Сохранен 587
https://2ch.hk/sci/res/249033.html
24 декабря Архивач восстановлен после серьёзной аварии. К сожалению, значительная часть сохранённых изображений и видео была потеряна. Подробности случившегося. Мы призываем всех неравнодушных помочь нам с восстановлением утраченного контента!

Математика, тред 11

 Аноним 19/03/15 Чтв 21:36:59 #1 №249033 
14267902191490.jpg
Предыдущий обнуляется тут >>241679

Начнём с задачи: какова мощность множества биективных отображений из бесконечного множества в себя?
Аноним 19/03/15 Чтв 22:18:59 #2 №249053 
>>249033
>биективных отображений из бесконечного множества в себя
Это автоморфизмом называется.

>какова мощность
Такова же как и мощность множества всех функций из A -> A. То бишь, ӀAut(A)Ӏ = ӀAӀ^ӀAӀ.
Аноним 19/03/15 Чтв 22:24:10 #3 №249059 
>>249053
Ну хорошо, вот в случае счётного множества получится континуум, или больше?
Аноним 19/03/15 Чтв 22:28:46 #4 №249065 
>>249059
Континуум. N^N = 2^N = R.
Аноним 19/03/15 Чтв 22:35:31 #5 №249072 
>>249033
Гайс вкиньте годной литературы по коммутативной алгебре
Аноним 19/03/15 Чтв 22:39:21 #6 №249074 
>>249072
Ну, классика это Атья Макдональд, но там вся суть комм. алгебры в упражнениях забита.

Я недавно наткнулся на неплохую книгу, правда на английском, но для меня это норм.
David Eisenbud - "Commutative algebra with a view towards algebraic geometry".
Рекомендую.
Аноним 19/03/15 Чтв 22:44:34 #7 №249077 
>>249074
Благодарю, посмотрю на досуге
Аноним 19/03/15 Чтв 23:10:20 #8 №249082 
>>249072
Ебошь matsumura и будешь на всех как на говно смотреть
Аноним 19/03/15 Чтв 23:28:44 #9 №249090 
>>249059
А в случае континуального множества?
Аноним 19/03/15 Чтв 23:59:17 #10 №249101 
14267987570990.png
>>249090
Ну тут уже не так все однозначно. Если принять гипотезу континуума, то |R|= 2^ω = ω1, тогда то теореме с пика, ω1^ω1 = 2^ω1 = [CH] = ω2. В общем, алефу второму равно. Без гипотезы континуума - хуй знает.
Аноним 20/03/15 Птн 00:10:08 #11 №249104 
>>249033
Кто нибудь может подкинуть книг по континуальным интегральчикам?
Аноним 20/03/15 Птн 00:20:42 #12 №249109 
>>249104
Лучшие интегралы под водовку и картофанчик в 7 томах, издательство мехмата МГУ
Аноним 20/03/15 Птн 00:33:25 #13 №249113 
>>249109
Наелся и напился уже, хочу освоить некст левел
Аноним 20/03/15 Птн 06:18:36 #14 №249156 
Немецкий или французский в 2015?
Дискусс
Аноним 20/03/15 Птн 06:27:11 #15 №249158 
>>249156
китайский
Аноним 20/03/15 Птн 12:04:00 #16 №249207 
>>249156
Немецкий
Аноним 20/03/15 Птн 15:00:45 #17 №249268 
Есть 2 ортогональных вектора в R^2 с целочисленными координатами. Всегда ли площадь прямоугольника, образованного этими векторами будет целочисленной? Мне кажется, что всегда.
Аноним 20/03/15 Птн 15:09:27 #18 №249271 
>>249268
Чет я затупил, это очевидно, потому что площадь прямоугольника - это скалярное произведение этих векторов.
Аноним 20/03/15 Птн 15:11:38 #19 №249272 
>>249271
векторное точнее
Аноним 20/03/15 Птн 16:39:18 #20 №249297 
>>249272
>площадь прямоугольника
>векторное произведение
>прямоугольника
Аноним 20/03/15 Птн 18:05:39 #21 №249307 
>>249297
Че, в блоге тифарета про это не пишут?
Аноним 20/03/15 Птн 18:56:41 #22 №249313 
>>249268
>>249271
Вектора {1, 1} и {-2, 2}. Скалярное произведение 1 -2 + 1 2 = 0, ортогональны. Достраивай до прямоугольника и считай площадь.
Аноним 20/03/15 Птн 19:45:54 #23 №249316 
>>249033
Как изучить нормальную математику? Я имею в виду топосы, когомологии пучков итд. Времени просто не хватает
Аноним 20/03/15 Птн 19:49:43 #24 №249317 
>>249313
4
Аноним 20/03/15 Птн 20:54:06 #25 №249329 
>>249313
А косо умножить, не?
Аноним 20/03/15 Птн 20:55:32 #26 №249330 
>>249271
Псевдоскалярное.
Аноним 20/03/15 Птн 23:37:21 #27 №249361 
>>249316
Почаще заходи в этот тред. В какой-то области быстрее всего можно начать ориентироваться следя за содержанием и стилем дискуссий настоящих мастеров, которые происходят как раз тут.
Аноним 20/03/15 Птн 23:37:39 #28 №249362 
>>249316
Почаще заходи в этот тред. В какой-то области быстрее всего можно начать ориентироваться следя за содержанием и стилем дискуссий настоящих мастеров, которые происходят как раз тут.
Аноним 21/03/15 Суб 00:08:33 #29 №249366 
Вопрос образования. Допустим, есть этакий undergraduate студент с базовым знанием счисления и линейной алгебры/аналитической геометрии. Сам вопрос: как из него с помощью русскоязычных книг сделать чистого прикладника-технаря? Вот что я предлагаю:

1. Углубить знания в мат. анализе: Зорич (какое издание лучше всего?).
2. Прикладной аспект в линейной алгебре (теорема Жордана и т.д.).
http://rutracker.org/forum/viewtopic.php?t=3103883
(Дополнительные главы линейной алгебры Беклемишева)
3. Теория вероятностей и мат. стастистика
http://rutracker.org/forum/viewtopic.php?t=2808181
(Курс Смирнова и Дунина-Барковского)
4. Дифференциальные уравнения
http://rutracker.org/forum/viewtopic.php?t=2461035
(Курс Эльсгольца)
Еще что-нибудь прикладное по диф. уравнениям?
5. "Элементы прикладной математики" Мышкиса.
6. Функциональный анализ
(Элементы теории функций и функционального анализа Колмогорова)
7. Топология (Лекции по топологии для физиков Шапиро)
8. Теория хаоса (хуй знает, Кроновер?)
9. Мат. моделирование (Самарский?)
10. Численные методы (???)
11. "Дискретная математика для инженера" Кузнецова.
Аноним 21/03/15 Суб 12:44:36 #30 №249436 
>>249366
А что такое прикладной математик? Правда ли, что в современном мире он неотрывно связан с теорией алгоритмов? Не понимаю, как можно быть прикладником и не считать что-то на ЭВМ, а это без знания алгоритмики и структур данных кажется будет фейловато получаться.
Аноним 21/03/15 Суб 13:09:14 #31 №249440 
>>249436
>Не понимаю, как можно быть прикладником и не считать что-то на ЭВМ, а это без знания алгоритмики и структур данных кажется будет фейловато получаться.
Вот тебе примитивный взгляд на вещи: если тебе надо построить регрессию по опытным данным, то тебе просто надо владеть мат. статистикой (понимать, что такое регрессия) и разбираться в синтаксисе и семантике математического пакета, в котором ты проводишь вычисления.
Аноним 21/03/15 Суб 13:28:17 #32 №249445 
>>249440
>разбираться в синтаксисе и семантике математического пакета
лол, при этом ни бум бум в теории алгоритмов?
Аноним 21/03/15 Суб 13:58:34 #33 №249452 
>>249090
Континуум.
Аноним 21/03/15 Суб 15:02:30 #34 №249457 
14269393508460.jpg
14269393508461.png
14269393508542.png
14269393508683.jpg
>>249436
>теория алгоритмов
>знание алгоритмики и структур данных
Аноним 21/03/15 Суб 19:18:37 #35 №249535 
>>249457
+15
Аноним 21/03/15 Суб 20:20:28 #36 №249558 
14269584288540.jpg
>>249033
Школьнику поможете? Никак не получается.
Аноним 21/03/15 Суб 20:22:46 #37 №249560 
>>249558
>>/un/
Аноним 21/03/15 Суб 20:47:09 #38 №249565 
>>249436
бамп вопросу
Аноним 21/03/15 Суб 21:21:35 #39 №249579 
>>249436
>Не понимаю, как можно быть прикладником и не считать что-то на ЭВМ
Исследовать вопросы, возникающие на практике - не то же самое, что брать конкретные данные и что-то считать. Вот скажем тот же алгоритм сортировки придумать - для этого же не надо брать ворох чисел и сортировать их. Но да, чаще всего прогать приходится, т.к. лучший способ проверить свои предложения - запрогать и посмотреть, как оно.
>а это без знания алгоритмики и структур данных кажется будет фейловато получаться.
Ну не обязательно все знать и помнить, понадобилось решать такую-то задачу - покопался в литературе на предмет наиболее быстрого алгоритма (более удобной структуры данных для хранения, ватевер), нашел, изучил, реализовал, радуешься. А еще лучше "нашел известный пакет, где это уже реализовано" вместо "реализовал".
Аноним 21/03/15 Суб 21:27:06 #40 №249580 
>>249558
Оно ж тупое как пиздец. Первое условие что у от -1 до 0. Второе - это квадратное уравнение по х. При заданных у и а у него одно решение будет лишь если это полный квадрат (дискриминант равен нулю). Получится квадратное уравнение по у - оттуда выйдет условие на а из того же дискриминанта. Какой мудак такие гроба считать заставляет?
Аноним 21/03/15 Суб 21:56:44 #41 №249593 
>>249580
> Какой мудак такие гроба считать заставляет?
Я когда в школе учился, весь 11 класс подобное считал. Мы типа к егэ готовились, лол.
Аноним 21/03/15 Суб 22:40:22 #42 №249608 
>>249558
ну тип первое уравнение банально на игрек дает ограничение в виде полосы
второе тупо однопараметрическое семейство окружностей
находишь такие значения параметра, когда окружность касается полосы и все, готово
Аноним 22/03/15 Вск 00:08:54 #43 №249616 
Посоветуйте хорошую литературу по р-адическим числам.
Аноним 22/03/15 Вск 00:50:46 #44 №249624 
Если комплексное число определяется как пара вещественных чисел, то как можно сравнивать комплексные и вещественные числа? Получается, что 2 = (2, 0), но это тупо, так не должно быть.
Аноним 22/03/15 Вск 00:54:29 #45 №249626 
>>249624
Если рациональное число определяется как пара целых чисел (с точностью до эквивалентности), то как можно сравнивать рациональные и целые числа? Получается, что 2 = (2, 1), но это тупо, так не должно быть.
Аноним 22/03/15 Вск 00:59:41 #46 №249628 
>>249624
как рациональные числа
Аноним 22/03/15 Вск 02:38:37 #47 №249636 
>>249624
Что значит "сравнивать"? Насколько я помню, на поле комплексных чисел нельзя установить отношения порядка.
Аноним 22/03/15 Вск 04:02:06 #48 №249659 
>>249636
Можно построить очевидный предпорядок, сравнивая модули комплексных чисел обычным порядком.
И можно сказать точно, что есть полный порядок, только далеко не факт, что его можно представить явно.
Аноним 22/03/15 Вск 10:06:45 #49 №249694 
>>249636
Согласованного с операциями? Просто порядок, очевидно, можно представить.
Аноним 22/03/15 Вск 12:10:38 #50 №249704 
Не, просто вот почему я это спросил: говорят, что R - подмножество C, то есть, если мы возьмем любое вещественное x, то существует комплексное z такое, что x = z, а именно, это будет число (x, 0). Но с точки зрения языка это как-то тупо, потому что x и (x, 0) - это разные объекты.

>>249626
> Если рациональное число определяется как пара целых чисел
Ну хз, в википедии не так определяется.
Аноним 22/03/15 Вск 12:26:04 #51 №249710 
В википедии написано следующее:
> Вещественные числа являются в этой модели подмножеством множества комплексных чисел и представлены парами вида (x,0).
Но тогда получается, что вещественное число - это пара двух вещественных чисел, а каждое число в этой паре - опять пара вещственных чисел и т.д.

Если определить C как множество чисел вида a + bi, то эта проблема решается, но возникает проблема с определением i. Если определить i как sqrt(-1), то получается какой-то крайне таинственный объект. Такое определение мне нравится еще меньше.
Аноним 22/03/15 Вск 12:33:34 #52 №249712 
Есть идея определить R' = { (x, 0) | x принадлежит R } и если действовать в R' по правилам C, то получится, что у R' те же свойства, что и у R. Тогда C = R2 и R' - подмножество C, все хорошо.

Тут опять есть неочевидный момент:
> если действовать в R' по правилам C, то получится, что у R' те же свойства, что и у R
Но вот в это я уже верю.
Аноним 22/03/15 Вск 13:06:50 #53 №249720 
Может ли групповое кольцо группы без кручения содержать делители нуля?
Аноним 22/03/15 Вск 13:55:02 #54 №249729 
>>249710
>Если определить i как sqrt(-1), то получается какой-то крайне таинственный объект.
Kek, с каких пор алгебру стала интересовать природа объектов? Есть элемент множества, у него есть свойств i* i= -1 . Множество с таким элементом является полем и будет расширением множества вещественных чисел, которое содержит все корни любого многочлена над R. Всё. И это более чем весомое обоснование введения поля с таким элементом.
Аноним 22/03/15 Вск 14:01:02 #55 №249731 
>>249272
>векторное произведение
>векторное
>в R^2
Аноним 22/03/15 Вск 14:10:15 #56 №249733 
>>249729
> Есть элемент множества
Какого?
Аноним 22/03/15 Вск 14:13:20 #57 №249735 
>>249733
Я к тому, что это элемент С, а если мы еще не определили С, то как мы можем определить умножение в С?
Аноним 22/03/15 Вск 14:16:47 #58 №249737 
>>249735
Ну хотя нет, на самом деле нормально все.
Аноним 22/03/15 Вск 16:06:10 #59 №249766 
>>249033
это тот старый хуй из интерстеллара?
Аноним 22/03/15 Вск 16:33:24 #60 №249769 
>>249766
Это Ацюковский.
Аноним 22/03/15 Вск 17:14:55 #61 №249772 
>>249636
Можно, конечно.
p = a+ib и q= x+iy - два комплексных числа.
p < q, если a < x или (a=x и b < y).
Например, 3+i10 < 5+i2; 7+i5 < 7+i9.
Аноним 22/03/15 Вск 17:20:53 #62 №249773 
>>249769
Какой, блядь, Ацюковский, если он американец по сюжету?
Аноним 22/03/15 Вск 19:55:11 #63 №249813 
>>249772
Иди водовки накати лучше.
Аноним 22/03/15 Вск 20:06:07 #64 №249816 
>>249772
Это отношение не выдерживает умножение:
-1 < 1
-i < i
1 < -1
Аноним 22/03/15 Вск 20:31:42 #65 №249820 
>>249816
Конечно. Но согласованность с арифметическими операциями и не задумывалась. Просто было сказано, что на множестве комплексных чисел вообще никакого порядка нельзя установить, вот я и привёл явную конструкцию одного из возможных порядков.
Аноним 23/03/15 Пнд 02:57:06 #66 №249932 
14270686268890.png
>>248829
Ну где же вы, эксперты по формам?
Аноним 23/03/15 Пнд 04:12:03 #67 №249939 
>>249033
Иван Фесенко тут теорию Мочидзуки осилил:
https://www.maths.nottingham.ac.uk/personal/ibf/notesoniut.pdf
Аноним 23/03/15 Пнд 04:18:03 #68 №249940 
ребят, я убогий школьник и молю о помощи:
lim((sqrt(5-x)-2)\(sqrt(2-x)-1)) при х->1. Хоть убейте, но выходит 2. Такая элементрный пример, но не получается. 1\2 по ответам. Даже в вольфрам альфа 1\2, всегда а не 2.
Аноним 23/03/15 Пнд 07:20:29 #69 №249950 
>>249932
classical stokes
Аноним 23/03/15 Пнд 08:42:58 #70 №249955 
Парни, есть закон при уменьшении числа слева в два раза, число справа уменьшается в на четверть. Я так понимаю это экспоненциальная функция, но слишком тупой что бы из этого закона понять какая конкретно. Поможешь?
Пример: 240 = 100
120 = 75
60 = 56.25
30=42.1875
sageАноним 23/03/15 Пнд 09:07:40 #71 №249959 
>>249940
lim((sqrt(5-x)-2)\(sqrt(2-x)-1)) при х->1
lim((sqrt(5-1)-2)\(sqrt(2-1)-1))
lim((sqrt(4)-2)\(sqrt(1)-1))
lim(2-2)\(1-1)) или lim(-2-2)\(-1-1))
lim(0\0) или lim(-4\-2)
lim(0\0) или lim(2)

Что не так?
Аноним 23/03/15 Пнд 11:38:06 #72 №249980 
14270998863110.jpg
Подскажите, где можно прочитать исчерпывающий, современно изложенный материал по задаче о каноническом виде пары линейных операторов, отысканию неразложимых объектов в категории представлений соответствующего колчана и прочем связанном?
Аноним 23/03/15 Пнд 12:03:14 #73 №249983 
>>249959
>или lim(-2-2)\(-1-1))
Откуда вот это? Там lim(0\0) и полная неопределенность получается.
Аноним 23/03/15 Пнд 12:35:56 #74 №249985 
>>249940
Может где скобку забыл?


И в тоже время объясните, что такое поле, кольцо. Только по-простому.
а то иногда встречается и теряется смысл.
вроде как поле - это просто числовая прямая может быть, т.е. где операции сложения, умножения заданы.
почему в алгебре так любят кольцами обмазоваться?
Аноним 23/03/15 Пнд 12:57:19 #75 №249992 
>>249985
Потому что полезных колец намного больше чем полей.
Аноним 23/03/15 Пнд 14:18:23 #76 №250002 
>>249992
Не "полезных", а "любопытных". Это разные вещи. Полезно только одно-единственное кольцо: R.
Аноним 23/03/15 Пнд 15:11:46 #77 №250014 
>>249033

Накидайте годных книг по основаниям математики - очень надо.
Аноним 23/03/15 Пнд 15:12:40 #78 №250015 
>>249766

Эт Кургинян из Донецка после ухода Стрелкова из Славянска.
sageАноним 23/03/15 Пнд 15:18:26 #79 №250017 
>>249033
Поясните вращение октонионов во всех полях.
Аноним 23/03/15 Пнд 15:26:51 #80 №250021 
>>249992

Много в ней колец, полей и рек...
Аноним 23/03/15 Пнд 15:27:53 #81 №250022 
>>249992
>>250002
ну так может вы всё-таки скажете по-простому, что такое кольцо и поле.
и в чём там фишка?
Аноним 23/03/15 Пнд 15:29:58 #82 №250023 
>>250022
https://ru.wikipedia.org/wiki/Кольцо_%28математика%29
Что в этом определении тебе не ясно?
Аноним 23/03/15 Пнд 15:38:12 #83 №250025 
>>250014

бамп вопросу
Аноним 23/03/15 Пнд 15:54:17 #84 №250027 
>>250023
ну вот вещественные числа, это что?
поле, кольцо?
почему в твоей ссылке и https://ru.wikipedia.org/wiki/%D0%9F%D0%BE%D0%BB%D0%B5_(%D0%B0%D0%BB%D0%B3%D0%B5%D0%B1%D1%80%D0%B0)
и там и там фигурируют те же структуры (вещественные, рациональные числа)

Насколько я мог понять из всего это ада непонятных терминов, то кольцо и поле отличаются количеством "операций" которые можно выполнять над элементами этих структур.
Хотя, в общем, они могут и совпадать.
Так, да?
Аноним 23/03/15 Пнд 15:55:19 #85 №250029 
>>250027

Ты лучше спроси что такое натуральные числа. ;)
Аноним 23/03/15 Пнд 16:15:41 #86 №250032 
>>250025
Очевидно Бурбаки
Аноним 23/03/15 Пнд 16:16:52 #87 №250033 
>>250032

У него по матлогике ничего нет.
Аноним 23/03/15 Пнд 16:20:08 #88 №250034 
>>249955
Никто не знает что ли?
Аноним 23/03/15 Пнд 16:29:42 #89 №250035 
>>249955
Если предположить, что функция имеет вид y = axb, подставить точки (240, 100) и (120, 75) и решить систему относительно a и b, получим
b = 0.41504
a = 10.283
Аноним 23/03/15 Пнд 16:40:10 #90 №250036 
>>250027
Операций там 2. Если ты посмотришь на определение кольца и поля и посмотришь, какие пункты там совпадают, станет ясно, что поле является кольцом, но кольцо необязательно является полем.
Аноним 23/03/15 Пнд 16:47:25 #91 №250037 
>>250027
>>250036
Алсо, это написано в википедии:
> Иерархия структур следующая:
> Коммутативные кольца ⊃ целостные кольца ⊃ факториальные кольца ⊃ области главных идеалов ⊃ евклидовы кольца ⊃ поля.
Аноним 23/03/15 Пнд 16:54:23 #92 №250039 
Что такое трансцендентное число? Объясните пожалуйста попроще. Т.е. там после запятой бесконечное число неповторяющихся знаков?
Аноним 23/03/15 Пнд 17:05:19 #93 №250040 
>>250039
Число, непредставимое натуральной дробью.
Аноним 23/03/15 Пнд 17:06:04 #94 №250041 
>>250002
Полезны куча колец, не являющиеся при том полями. При чём "полезны" в самом картофанном смысле, те же матрицы.
Аноним 23/03/15 Пнд 17:07:09 #95 №250042 
>>250039
В википедии все написано.
Аноним 23/03/15 Пнд 17:07:31 #96 №250043 
>>250040
Че, блять?
Аноним 23/03/15 Пнд 17:10:02 #97 №250044 
>>250039
>там после запятой бесконечное число неповторяющихся знаков
Нет, это иррациональное.

Алгебраическими числами над полем рациональных чисел называются числа, которые являются корнями некоторого многочлена с рациональными коэффициентами. Многочлена - значит, икс возводится только в неотрицательные целые степени.

Например, x^2 = 2. Корнем этого уравнения будет корень из двух. Корень из двух - алгебраическое число.

Трансцендентными называются числа, не являющиеся алгебраическими. Например, пи. Не существует такого многочлена с рациональными коэффициентами, что пи является его корнем.

Однако пи не является трансцендентным над полем вещественных чисел, так как пи очевидно является корнем многочлена x=пи лол.

>>250040
Вздор.
Аноним 23/03/15 Пнд 17:10:35 #98 №250045 
>>250041
Примеры пользы? Кольцо - польза, кольцо - польза.
Аноним 23/03/15 Пнд 17:14:51 #99 №250046 
>>250044
>трансцендентным над полем вещественных чисел
Возможные ли такие числа?
Аноним 23/03/15 Пнд 17:23:33 #100 №250048 
>>250044
>Однако пи не является трансцендентным над полем вещественных чисел, так как пи очевидно является корнем многочлена x=пи лол.
Погоди, во-первых Пи это вещественное число. Во-вторых х=пи это не многочлен, это простейший одночлен!
Аноним 23/03/15 Пнд 17:38:24 #101 №250050 
>>249959
Вот именно, что я делил все на (1-1). И выходило 2. еще выходило 1, когда (1-1) разбивал на (1+1)(1-1). Здесь 100% правильный ответ 1\2. Получил его подставив любое число из промежутка [0;1). Здесь что-то связанно с областью значений функции в знаменателе, только не могу понять, что именно.
Аноним 23/03/15 Пнд 17:40:36 #102 №250051 
>>249983
>>249983
откуда минусы у двойки и единицы тоже не понял совсем. Но выходить должно 1\2, это точно. И в учебнике в ответах и в программах такой ответ. Я просто не могу понять совсем, как его решать.
Аноним 23/03/15 Пнд 17:42:48 #103 №250052 
>>250045
Надеюсь ты не про пользу матриц спрашиваешь.
Разнообразные множества функций с операцией поточечного умножения не всегда являются полями - возникают везде, в анализе, в геометрии. Конечные кольца вычетов - элементарная теория чисел. Ну итд.
Аноним 23/03/15 Пнд 17:46:33 #104 №250053 
>>250014

bump
Аноним 23/03/15 Пнд 17:54:27 #105 №250054 
>>250051
Если ты тоже не понял, то нахуя это написал? Но тут все правильно. Sqrt(4) можно представить как 2^2 либо -2^2.
Аноним 23/03/15 Пнд 18:02:02 #106 №250055 
>>250014
Френкель - Основания теории множеств
OMEGUS MAXIMUS !36qBqAlois 23/03/15 Пнд 18:17:31 #107 №250059 
14271238518550.jpg
Студентота тихонько заходит в тред.
Есть один курсач. По диффурам.
Задание что-то вроде "построить интегральные кривые дифференциальных уравнение с обоснованием существования, единственности, продолжаемости и т.д." и даны несколько уравнений, довольно простых, первого порядка, кажется все разделяющиеся.

Прошу насоветовать годных, достаточно доходчивых учебников.
Аноним 23/03/15 Пнд 18:40:03 #108 №250065 
>>250054
как корень квадратный из 4, может быть равен -2? я понимаю, что х во второй может быть и -2 и 2 для 4. но ты же не число а находишь(то, что под знаком корня), а ИЗВЛЕКАЕШЬ корень. Для положительного числа-положительный корень.Да и к тому же я сказал уже. 2 - ответ не правильный. Должно получиться 1\2.
Аноним 23/03/15 Пнд 18:41:06 #109 №250066 
>>250014
Бертран Рассел
Аноним 23/03/15 Пнд 18:45:32 #110 №250069 
>>250059
Ты что, не знал? Это же картофанчик. Бросай свой заборостроительный и ныряй в алгебраическую топологию с головой.
Аноним 23/03/15 Пнд 18:57:53 #111 №250074 
>>250059
Арнольд ОДУ
Аноним 23/03/15 Пнд 19:17:58 #112 №250076 
>>250065
>как корень квадратный из 4, может быть равен -2?
Ну ебана, тогда тебе нужно брать учебник за 5 класс школы (или в каком там классе начинают корни проходить?)
https://ru.wikipedia.org/wiki/Корень_(математика)
>Как видно из первого примера, у вещественного корня могут быть два значения (положительное и отрицательное), и это затрудняет работу с корнями. Чтобы обеспечить однозначность, вводится понятие арифметического корня, значение которого всегда неотрицательно, в первом примере это число 3.
У тебя в условии не сказано, что корень арифметический
Аноним 23/03/15 Пнд 20:08:24 #113 №250109 
>>250076
а что, для тебя sqrt(5-x) вещественный корень? или ты не различаешь число(sqrt(4)) от комплекскного числа? ты лучше попробуй найти 1\2.
Аноним 23/03/15 Пнд 20:08:32 #114 №250110 
14271305123800.jpg
>>250065
>>250051
>>249959
>>249940
Скажи спасибо, что мне сейчас нехуй делать. Все подробно тебе расписал (подробней некуда). Если просто подставлять единицу, то получается неорпеделенность 0/0. Поэтому тут нужно применять правило Лапиталя. Вообщем пирилейтед, если что не понятно поясню.
Аноним 23/03/15 Пнд 20:27:56 #115 №250116 
Фурье от exp(-x^2) - тоже гауссова функция (если ширина правильная). Похоже, функций с таким свойством (преобразование Фурье - такая же функция, как начальная ) может быть много разных. Есть ли какая-нибудь литература по поводу?
Аноним 23/03/15 Пнд 20:50:07 #116 №250127 
>>250109
Ну же школьник, ну куда ты пропал? наверняка в доту ушел играть, падла. Мне уже не терпится тебе все объяснить по поводу этого >>250110 простенького примера.
Аноним 23/03/15 Пнд 20:52:11 #117 №250129 
>>250066

Он же морально устарел, насколько я помню его программа обоснования эпично зафейлилась после теорем Гёделя.

>>250055

У него в книжке матлогики нет.
Аноним 23/03/15 Пнд 20:53:52 #118 №250131 
>>250110
благодарю, анон. Я здесь. я уже все давно решил БЕЗ правила лопиталя. У меня и до этого получалось найти предел 0\0 путем упрощений. Я же не учил еще, блядь, производных и т.д и не знаю ваши теоремы.
Аноним 23/03/15 Пнд 21:11:05 #119 №250133 
14271342655780.jpg
>>249033
помогите вспомнить как решать первое задание, с объяснением для дочки)
Аноним 23/03/15 Пнд 21:24:56 #120 №250136 
>>250133
Ну напиши как ты пытался решить это задание.
Аноним 23/03/15 Пнд 21:27:55 #121 №250138 
>>250136
осталось узнать формулу параболы которая проходит через две точки, вершину и рандомную точку.
Аноним 23/03/15 Пнд 21:31:49 #122 №250140 
>>250129
Не будет тебе строгого учебника по основаниям математики, такой области нет. Есть только полуфилософская и полуматематическая литература, вроде Френкеля. Не устраивает его строгость изложения, можешь читать параллельно Чёрча - Введение в математическую логику. Сам Френкель настоятельно её рекомендует.
Аноним 23/03/15 Пнд 21:32:27 #123 №250141 
>>250133
Решил. Но вы же знаете правила, папаша? Выложу и объясню все подробно, если предоставите пруфы сисек дочки с супом.

Аноним 23/03/15 Пнд 21:34:33 #124 №250143 
>>250141
Тогда не нужно.
Аноним 23/03/15 Пнд 21:34:37 #125 №250144 
>>250116
>Похоже, функций с таким свойством (преобразование Фурье - такая же функция, как начальная )

С чего ты взял?
Аноним 23/03/15 Пнд 21:36:24 #126 №250145 
>>250140

У Чёрча не самый лучший учебник по матлогике, он там тоже особо не заморачивается строгостью изложения.
Аноним 23/03/15 Пнд 21:38:39 #127 №250146 
>>250140

А кроме Френкеля есть что еще? Пущщай полуфилософская.
Аноним 23/03/15 Пнд 21:39:26 #128 №250148 
14271359666930.jpg
Помогите узнать формулу параболы которая проходит через две точки, вершину и рандомную точку. В первом задании.
Аноним 23/03/15 Пнд 21:47:47 #129 №250152 
>>250148
Поздно! Если бы сразу написал, что ты жалкий хикка и тебе больше не на кого расчитывать, то другое дело. А теперь либо сиськи дочки, либо уебывай!
OMEGUS MAXIMUS !36qBqAlois 23/03/15 Пнд 21:52:26 #130 №250154 
14271367467840.jpg
>>250074
Благодарю, попробую осилить за оставшееся время.
Аноним 23/03/15 Пнд 21:56:41 #131 №250156 
>>250152
>сиськи дочки
Харви залогинься
Аноним 23/03/15 Пнд 23:14:50 #132 №250168 
>>250148
Это в шкалке несобственные интегралы сейчас проходят? Или это госы?
Аноним 23/03/15 Пнд 23:31:20 #133 №250171 
>>249980
http://halgebra.math.msu.su/alg-geom/
Аноним 24/03/15 Втр 00:18:15 #134 №250180 
>>250014
Столл.
Аноним 24/03/15 Втр 02:18:04 #135 №250193 DELETED
Может кто-нибудь читал заметки математиков о том, как стать математиком? Натыкался на учебную программу, составленную Михаилом Вербицким, и еще читал что-то вроде биографии и студенческих, научных переживаний Яу Шинтана. Читал расплывчатое интервью Романа Михайлова. У кого-нибудь есть что по теме?
Аноним 24/03/15 Втр 07:36:09 #136 №250211 
>>250193
Хуйцы почаще посасывай, а математическое знание, оно придёт. Все известные математики были геями.
Аноним 24/03/15 Втр 08:07:45 #137 №250217 
>>250211
Удваиваю.
Аноним 24/03/15 Втр 09:16:42 #138 №250230 
>>250180

Полистал немного, логику он совсем поверхностно излагает, и сидит на наивной теории множеств. Нехотет такие основания.
Аноним 24/03/15 Втр 09:24:26 #139 №250232 
>>250230
Марио не может осилить ничего сложнее инструкции "press A to jump".
Аноним 24/03/15 Втр 10:29:19 #140 №250259 
>>250230
>Пущщай полуфилософская.
>он совсем поверхностно излагает, и сидит на наивной теории множеств. Нехотет такие основания.
Съеби отсюда.
Аноним 24/03/15 Втр 11:36:38 #141 №250267 DELETED
>>250211
Я, кстати, последние пару дней мастурбировал на фотографии членов, серьезно переживал по этому поводу: не хочу, чтоб пидором называли. А оказывается - может оно и к лучшему. Но все равно, хотелось бы почитать что-нибудь о жизненном пути современных математиков.
Аноним 24/03/15 Втр 11:42:08 #142 №250268 
>>250259

Хуй тебе в рыло.

>>250232

Что за бред.
Аноним 24/03/15 Втр 11:43:09 #143 №250269 
>>250193

Миша гнилой пидорас, не стоит ему верить.
Аноним 24/03/15 Втр 11:46:43 #144 №250271 DELETED
>>250269
Первые пару курсов у него замечательно расписаны + он написал отличный учебник для матшкольников.
Аноним 24/03/15 Втр 11:47:48 #145 №250272 
>>250271

Школопроблемы.

PS Почитай его твитор - охуеешь. Это просто новодворская с хуем.
Аноним 24/03/15 Втр 11:48:36 #146 №250273 
>>250014

Плюсую вопрос.
Аноним 24/03/15 Втр 11:59:39 #147 №250277 
>>250267
>что-нибудь о жизненном пути современных математиков
http://vestnik-old.samsu.ru/articles/17/.pdf
Аноним 24/03/15 Втр 12:00:56 #148 №250280 
>>250273
>>250268
Сёма, уходи. Тебе тут не рады.
Аноним 24/03/15 Втр 12:01:47 #149 №250283 
>>250280

За себя говори, МРАЗЬ!
Аноним 24/03/15 Втр 12:03:44 #150 №250284 DELETED
>>250277
спасибо, братан
Аноним 24/03/15 Втр 12:09:52 #151 №250288 
>>250046
Бамп вопросу.
Аноним 24/03/15 Втр 12:11:05 #152 №250290 
>>250288

Сёма, уходи. Тебе тут не рады.
Аноним 24/03/15 Втр 12:19:36 #153 №250292 
>>250288
Нет. Потому что любое трансцендентное число вещественно.
Аноним 24/03/15 Втр 12:21:27 #154 №250293 
>>250290
>>250283
Буйство семёна итт.
Аноним 24/03/15 Втр 12:59:31 #155 №250299 
14271911710870.gif
Помнится тут был анон, который утверждал, что теория категорий может объяснить почему сопряжение сопряжения линейного пространства эквивалентно самому пространству. Собственно вопрос - почему? И почему тогда сопряжение пространства не эквивалентно самому пространству?
Аноним 24/03/15 Втр 13:22:02 #156 №250303 
>>250299
Тут вся суть в ЕСТЕСТВЕННОСТИ
Аноним 24/03/15 Втр 14:24:07 #157 №250309 
>>250299

Нет, утверждалось о несводимости теории множест к теоркату.
Аноним 24/03/15 Втр 14:38:53 #158 №250312 
>>250277
>возраст не решает
А ведь возраст решает почти все. С возрастом ухудшается рабочая память, а с ней и способность к успешному разбору проблем.
Аноним 24/03/15 Втр 16:12:13 #159 №250334 
>>250046
Ответ зависит от того, что считать числом. Просто трансцендентные расширения возможны, конечно. Например, рассмотри совокупность рациональных функций от одной переменной x с действительными коэффициентами. Она будет полем и, очевидно, расширением поля действительных чисел. x будет трансцендентным элементом над R. Это поле неархимедово, в нём существуют актуальные бесконечно-малые и бесконечно-большие.
Аноним 24/03/15 Втр 17:49:15 #160 №250394 
14272085556890.jpg
14272085556901.jpg
Пацаны, не бейте школьника. Короче, есть неравенство пикрелейтед. Вроде бы решение правильное, но ответ не такой, а тот, что справа. Насколько я понял, я проебался с возведением в квадрат. Так вот, как нужно правильно решать подобные вещи, чтобы не проебываться?
На другой пикче то, что было сначала.
Аноним 24/03/15 Втр 18:08:50 #161 №250400 
>>250394
Ты не учел, что выражение под корнем должно быть неотрицательным.
Аноним 24/03/15 Втр 18:37:09 #162 №250418 
>>250400
Да вроде учел, в одз тип. Там получилось, что x принадлежит от минус бесконечности до -1, ну и от 2 до плюс бесконечности. А ответ типа от минус бесконечности до -2. Если икс равен -1, то неравенство не выполняется.
Аноним 24/03/15 Втр 18:38:10 #163 №250419 
>>250418
Хотя выполняется, лол. А че ответ другой
Аноним 24/03/15 Втр 18:39:11 #164 №250422 
>>250419
Блять я мудак, не выполняется
Аноним 24/03/15 Втр 18:42:14 #165 №250424 
>>250400
Все понял, я неправильно корни нашел. Не обращайте на меня внимания. Спасибо
Аноним 24/03/15 Втр 19:57:32 #166 №250454 
>>250312
С возрастом появляется возможность работать. А до этого молодость, телки, нехватка денег и прочее говно.
Аноним 24/03/15 Втр 20:00:12 #167 №250455 
>>250454
> 2ch.hk
> молодость, телки
ну-ну
Аноним 24/03/15 Втр 20:06:47 #168 №250461 
>>250455
Ну если хиккой сидеть дома и математиком не станешь. Так то у всех тру хикк типа Перельмана были и кафедры и математические круги, и бабла он заработал в америке, чтобы из дома не выходить.
Аноним 24/03/15 Втр 20:15:48 #169 №250472 
>>250461
> молодость, телки
> кафедры и математические круги
Действительно, одно и то же.
Аноним 24/03/15 Втр 20:42:12 #170 №250485 
>>250277
Моар есть?
Аноним 24/03/15 Втр 21:09:53 #171 №250494 
>>250272
У него нет твиттера лол. Он принципиально кормится только в своей лжр.
Аноним 24/03/15 Втр 21:31:31 #172 №250509 
>>250494

Зачем ты пиздишь?

https://twitter.com/misha_verbitsky
Аноним 24/03/15 Втр 21:32:31 #173 №250511 
>>250509
https://vk.com/doc137862366_374777132?hash=19d358996e1313d12e&dl=77218afb0c313949a4
Годнота.
Аноним 24/03/15 Втр 21:33:52 #174 №250513 
>>250511

Хуйня. Логику он слишком поверхностно излагает, нормальных оснований математики не строит.
Аноним 24/03/15 Втр 21:35:00 #175 №250514 
Вообще мишка переоцененный антирусский петукх.
Аноним 24/03/15 Втр 21:37:29 #176 №250516 
>>250513
Это не он, это его студенты, это ваще другая программа, вообще о другом.

>Хуйня. Логику он слишком поверхностно излагает, нормальных оснований математики не строит.
Потому что в курсе алгеобраической геометрии это нахуй не нужно. Да и в принципе не нужно, это даже не математика.
Аноним 24/03/15 Втр 21:41:53 #177 №250521 
>>250516

Логика и метаматематика это все части математики, т.к. без них математики не существует.

Аноним 24/03/15 Втр 21:44:29 #178 №250523 
>>250521
Нет, это не части математика, это абсолютно независимые области, связанные только между собой.
Аноним 24/03/15 Втр 21:48:36 #179 №250525 
>>250523

Это потому, что ты дурак и не видишь откуда происходят основные понятия и что любое доказательство использует логические средства, строго по определению доказательства. Ты думаешь раз в твоих букварях ничего этого нет и с первых страниц херачат матан, то ничего кроме этого в к математике не относится. Но это это только потому, что ты хуйло тупое ничего не понял, а не из-за того что так на самом деле.
Аноним 24/03/15 Втр 22:13:49 #180 №250530 
14272244291230.jpg
>>250525
Толсто. Попробуй ещё.
Аноним 24/03/15 Втр 22:20:28 #181 №250532 
>>250509
http://lj.rossia.org/users/tiphareth/1804746.html
Аноним 25/03/15 Срд 07:07:50 #182 №250593 
>>250530
Где тебе толсто?
Аноним 25/03/15 Срд 09:16:55 #183 №250617 
>>250532

В соответствии с требованиями законодательства доступ к запрашиваемому Интернет-ресурсу
закрыт.

А вообще, какая разница, это ж не фейк. Какая нибудь тупая пизда постит одну и ту же хуйню в десяток соцсетей, но разве это что-то меняет?
Аноним 25/03/15 Срд 09:18:00 #184 №250619 
>>250530

Толсто в анусе твоей мамки-шлюхи, когда я засовываю туда свои 20 см.
Аноним 25/03/15 Срд 09:27:26 #185 №250621 
>>250617
> В соответствии с требованиями законодательства доступ к запрашиваемому Интернет-ресурсу закрыт.
Я через тор захожу.
Аноним 25/03/15 Срд 09:43:12 #186 №250625 
>>250617
>В соответствии с требованиями законодательства доступ к запрашиваемому Интернет-ресурсу закрыт
Ебать ты лох.

Аноним 25/03/15 Срд 10:03:53 #187 №250629 
>>250621
>>250625
> я сам себе злобный буратина, обходящий защиту меня от террористов
Да вы же просто номинанты на премию Дарвина.
Аноним 25/03/15 Срд 10:06:01 #188 №250630 
>>250629
Лолшто.
Аноним 25/03/15 Срд 10:07:55 #189 №250631 
>>250630
Если перед оврагом поставят табличку "не подходить, опасно", то ты обойдёшь и прыгнешь в овраг?
Аноним 25/03/15 Срд 10:10:43 #190 №250632 
>>250621

Лол. Корячиться ради тупых мишкиных высеров? Да вы мазохист.
Аноним 25/03/15 Срд 10:15:20 #191 №250634 
>>250632
В интернете есть довольно много сайтов, которые заблочены и на которые приходится заходить через тор
Аноним 25/03/15 Срд 10:15:51 #192 №250635 
>>250634

Я про мишку говорю.
Аноним 25/03/15 Срд 10:22:30 #193 №250637 
>>250631
Что ты несешь, поехавший.
Аноним 25/03/15 Срд 10:23:32 #194 №250639 
>>250629
>>250631
Такая то жирнота.
Аноним 25/03/15 Срд 12:55:10 #195 №250705 
Поясните ребята, а где-нибудь на производстве хоть сколь элементарная алгебра применяется? Хочу закосить устроившись в оборонное предприятие, может есть возможность и чему-то действительно научится?
Аноним 25/03/15 Срд 14:41:51 #196 №250728 
>>250635
Тор-бразуер скачивается и устанавливается 2 минуты, с этим справится даже домохозяйка. И роскомнадзор дохуя чего банит. В какой-то период времени 4чан был забанен, блог овального забанен и т.д.
Аноним 25/03/15 Срд 15:22:29 #197 №250743 
14272861493320.jpg
>>250299
Ну что же вы, Гротендики?
Аноним 25/03/15 Срд 16:17:42 #198 №250764 
>>250728

Я на работе блокировку инета тором обхожу. Но ради гнилого пидораса мишки лень даже иконку клиткать и ждать загрузку.
Аноним 25/03/15 Срд 17:27:53 #199 №250783 
А чем собственно Мишка насолил кровавому режиму?
Аноним 25/03/15 Срд 17:29:26 #200 №250786 
>>250783

Это мишку надо спросить, чем ему режим насолил.
Аноним 25/03/15 Срд 17:42:11 #201 №250788 
>>250786
http://lenta.ru/articles/2012/04/13/boroda/
Например.
OMEGUS MAXIMUS !36qBqAlois 25/03/15 Срд 21:14:24 #202 №250889 
14273072645900.jpg
Как решается x^3/y^6=1 и x^3/y^6=-1 и как будут выглядеть графики?
На вольфраме проверил и вид графиков совпал с моим предположением, но всё равно не могу понять. Ёбанные корни, ненавижу, они плодят слишком много сущностей.
sageАноним 25/03/15 Срд 21:22:11 #203 №250891 
>>250889
Узнаю картинку. Неужели шизика почитываешь?
Аноним 25/03/15 Срд 21:53:40 #204 №250896 
>>250788

Гламурный петушок допизделся, лол. Сам виноват, нечего было бороду оскорблять. Хотели правовое государство? Так вот это оно.
Аноним 25/03/15 Срд 21:54:18 #205 №250897 
>>250891

Бар-Хиллела.
Аноним 25/03/15 Срд 23:02:29 #206 №250909 
Ну што, масематики, разобрались уже с множеством, которое содержит все множества, которые не содержат себя?
Аноним 26/03/15 Чтв 00:52:36 #207 №250927 
>>250889
У тебя ошибка в условии. Если вычесть второе уравнение из 1, получится 0 = 2.
Аноним 26/03/15 Чтв 00:53:32 #208 №250928 
>>250909
Гёделя наверни.
OMEGUS MAXIMUS !36qBqAlois 26/03/15 Чтв 01:21:59 #209 №250931 
14273221195320.jpg
14273221195561.jpg
>>250074
Ожидаемо практически ничего не осилил и вообще почти сутки спал.
Ладно, надеюсь анон будет няшей и поможет мне немного. Взамен могу написать что-нибудь на листе кровью по реквесту.

У меня есть примеры решения пары других курсовых и так почему-то используется в качестве критерия существования и единственности что-то вроде:
для dy/dx=f(x,y) (во всех заданиях 1-4 переменные разделяются так что даже dy/dx=f(x)g(y) ) - если частные производные d( f(x,y) )/dy и d( 1/f(x,y) )/dx существуют (и непрерывны?) на C (понятное дело C - подмножество R^2) то на C существует и единственно решение задачи Коши для любых (x',y') принадлежащих C.

Такой критерий верен? Не встречал такого в учебниках.
OMEGUS MAXIMUS !36qBqAlois 26/03/15 Чтв 01:25:29 #210 №250932 
>>250931
>и там почему-то
fix
OMEGUS MAXIMUS !36qBqAlois 26/03/15 Чтв 03:36:15 #211 №250947 
14273301757150.jpg
>>250931
>если частные производные d( f(x,y) )/dy ИЛИ d( 1/f(x,y) )/dx существуют (и непрерывны?) на C (понятное дело C - подмножество R^2) то на C существует и единственно решение задачи Коши для любых (x',y') принадлежащих C.
Фикс. Т.е. условие существования и единственности может не выполнятся там где обе эти производные не существуют или претерпевают бесконечный разрыв.
Аноним 26/03/15 Чтв 07:19:20 #212 №250955 
>>250947
сфотай анус жопы, Андрей :3
Аноним 26/03/15 Чтв 09:06:32 #213 №250960 
>>250014

Удваиваю вопрос.
Аноним 26/03/15 Чтв 14:18:53 #214 №251022 
Подскажите хорошую книгу по матану чтобы вникнуть и прочувствовать всё, уравнения, формулы, небо и даже Аллаха. Темы: дифференциальные уравнения, интегральчики какие-нибудь, комплексные числа, матрицы
Аноним 26/03/15 Чтв 14:35:16 #215 №251032 
>>251022

На русском только Кудрявцев, остальные (вроде Фихтенгольца) застряли на довоенном уровне.
Аноним 26/03/15 Чтв 14:40:58 #216 №251034 
>>251032
Спасибо, сейчас попробую. Добра.
Аноним 26/03/15 Чтв 15:10:52 #217 №251043 
>>250525
Ты понимаешь, что такое связанные области, долбоеб? Я могу исследовать что-то с помощью логики? Нет. Я лишь могу исследовать саму логику, т.е это отдельная область.
Аноним 26/03/15 Чтв 15:18:55 #218 №251046 
>>251043
>Я могу исследовать что-то с помощью логики? Нет.
Оно и видно.
Аноним 26/03/15 Чтв 15:20:03 #219 №251047 
>>251043
> Я могу исследовать что-то с помощью логики? Нет.
По-твоему, логика - это что-то типа абстрактной игры в домино, а не инструмент исследования принципов мышления и физической реальности? Дурачок.
Аноним 26/03/15 Чтв 17:08:24 #220 №251064 
>>250521
ловите школьника-максималиста
OMEGUS MAXIMUS !36qBqAlois 26/03/15 Чтв 17:24:26 #221 №251071 
14273798664220.jpg
>>250947
Эх анон, анон. Почему ты не можешь мне помочь?
Аноним 26/03/15 Чтв 17:30:28 #222 №251072 
>>251071

Что делает клещ?
Аноним 26/03/15 Чтв 17:31:13 #223 №251073 
>>251071

Да тут полтора школьника сидит, математиков нет нифига. Если люди даже логику не признают, о чём тут говорить.
Аноним 26/03/15 Чтв 17:50:37 #224 №251079 
Реквестирую годных книг по основам теории вероятностей. Давай анон, подскажи. Хочу изучить хотя бы основы предмета и ссать на лицо всем лудоманам с пруфами!
Аноним 26/03/15 Чтв 17:54:58 #225 №251080 
>>251071
Извиняй, няша, подзабыл я уже диффуры. Вот матан помню неплохо, линейную алгебру, аналитическую геометрию, а вот диффуры уже почему то напрочь стерлись из памяти. Хотя не сказать, что препод у нас по ним был очень плох, скорее наоборот. Хуй знает почему так.
Аноним 26/03/15 Чтв 17:55:56 #226 №251081 
>>251079

Ширяев, Вероятность.
Аноним 26/03/15 Чтв 17:56:41 #227 №251082 
>>251080

Тоже самое. Почему-то душа не лажала к дифурам, сдал и забыл.
Аноним 26/03/15 Чтв 18:15:04 #228 №251087 
>>251079
Хеннекен Теория вероятности и некоторые её приложения
Аноним 26/03/15 Чтв 19:40:36 #229 №251099 
>>249580
>>249608
Спасибо, завтра утром попробую. Мы диагностическую работу по подготовке к ЕГЭ писали. Хотел дома сам сделать, но нихуя не получалось. Это необязательный номер был, не литсей же.
Аноним 26/03/15 Чтв 19:55:34 #230 №251104 
>>250299
ТОЛКУ ОТ ВАС КАК ОТ ГОВНА, ПИДОРАСЫ ЕБАНЫЕ
OMEGUS MAXIMUS !36qBqAlois 26/03/15 Чтв 20:04:53 #231 №251109 
14273894939670.jpg
Является ли функцией одна точка?
Можно ли сказать что решением диффура dy/dx=-sinx/cosy является параметрически заданная функция {y=PI/2; x=PI}? При этом никакой интегральной кривой через эту точку, как я понял, не проходит.
Аноним 26/03/15 Чтв 20:30:34 #232 №251113 
>>251104
Полегче, тут тебе никто ничего не должен.
Аноним 26/03/15 Чтв 21:00:57 #233 №251121 
>>251109
>является параметрически заданная функция {y=PI/2; x=PI}
Очевидно же, что нет. Чтобы быть решением, нужно, чтобы существовала производная, а какая может быть производная у одной точки?
Аноним 26/03/15 Чтв 21:39:27 #234 №251126 
>>250299
Ты достал уже скулить, так что придётся объяснить. Рассмотрим категорию конечномерных векторных пространств над фиксированным полем. Рассмотрим два функтора из неё в себя: первый тождественный, а второй функтор каждому векторному пространству сопоставляет его второе сопряженное, а морфизмам, разумеется вторые сопряжения морфизмов. Зададим сопоставление: каждому векторному пространству V сопоставим изоморфизм f:V->V, f(v)(g)=g(v). Тогда это сопоставление является ЕСТЕСТВЕННЫМ ПРЕОБРАЗОВАНИЕМ тождественного функтора и функтора второго сопряжения. Поэтому говорят, что тот изоморфизм, который мы сопоставили, является естественным (иногда говорят каноническим).
Аноним 26/03/15 Чтв 21:40:39 #235 №251127 
>>251126
Опечатка, там f действует из V в второе сопряженное V
Аноним 26/03/15 Чтв 21:45:28 #236 №251130 
>>251079
Есть короче книжка Гмурмана, в ней все как для даунов написано. Из нормального я планирую вот это читать
http://dabirbook.com/uploadedfiles/files/1/book/fe286488cbb07c0b763370b82e6aa45a.pdf
, правда не сейчас, а где-то через год, потому что мне пока еще не хватает mathematical maturity.
Аноним 26/03/15 Чтв 22:22:34 #237 №251144 
>>250014

Бамп.
Аноним 26/03/15 Чтв 23:26:01 #238 №251161 
>>251047
Для меня логика это
>инструмент исследования принципов мышления
поэтому к математике она не имеет никакого отношения.
Аноним 26/03/15 Чтв 23:37:15 #239 №251163 
>>251047
>>251161
>Гвозди - это инструмент исследования молотка
Пиздец, каких только клованов не встретишь в жизни.
Аноним 27/03/15 Птн 00:13:48 #240 №251179 
>>251126
Ты не пользуешься тем, что пространство конечномерное векторное, а значит ты хуй и как минимум не умеешь объяснять.
Аноним 27/03/15 Птн 00:18:19 #241 №251184 
>>251179
>сопоставим изоморфизм
Вот здесь и воспользовался.
Я рассчитывал, что вопрошающий знает алгебру хотя бы в объёме первых двух курсов обычной программы универа.
Аноним 27/03/15 Птн 02:25:43 #242 №251229 
>>251163
Неправильная аналогия. Гвозди и молоток существуют в реальности.
Аноним 27/03/15 Птн 02:32:48 #243 №251231 
>>251229
Тогда вот тебе новая аналогия.
Логика - послушная лошадка. Куда запряг, туда и вывезет.
Аноним 27/03/15 Птн 02:33:05 #244 №251232 
>>251161
> Для меня логика это инструмент исследования принципов мышления
> поэтому к математике она не имеет никакого отношения.
Математика - исключительно мыслительная деятельность. См. статью "философия математики" в википедии.
Аноним 27/03/15 Птн 02:34:12 #245 №251233 
>>251231
> жиды придумали науку, чтобы запретить нам питаться еловыми шишками
Ясно.
Аноним 27/03/15 Птн 03:11:28 #246 №251237 
>>251232
Не знаю, зачем мне это, ведь это не имеет отношения к обсуждаемому вопросу. Я могу исследовать алгебру средствами геометрии? Могу. Могу применить теорию алгебру в анализе? Могу. А с логикой я ниче не могу, она сама по себе, это оторванная от математики область в себе.
Аноним 27/03/15 Птн 03:22:27 #247 №251238 
>>251237
Хочешь сказать, ты не можешь применять логику?
Аноним 27/03/15 Птн 03:25:14 #248 №251239 
>>251237
"Филосо́фия матема́тики — раздел философии науки, исследующий философские основания и проблемы математики: онтологические, гносеологические, методологические, логические и аксиологические предпосылки и принципы математики в целом, ее различных направлений, дисциплин и теорий."
Аноним 27/03/15 Птн 03:31:50 #249 №251240 
>>251237
Дурашка, ты говоришь не о качествах логики как науки, а о своём непонимании её.
Аноним 27/03/15 Птн 07:41:34 #250 №251262 
>>251237
А с логикой ты например можешь доказать адекватность (soundness) системы аксиом.
Аноним 27/03/15 Птн 08:45:56 #251 №251272 
Он прав, можно быть первоклассным математиком, ничего не зная о логике. Особенно доставляют (возникающие на почве комплекса неполноценности?) заявления вроде «логические аксиомы — самое главное в математике», «если обнаружится противоречие в строках символов математику можно отправлять на свалку», и, наконец, «вы просто не знаете логику», что говорит само за себя.
Аноним 27/03/15 Птн 09:39:03 #252 №251279 
>>251272

Если выяснится, что математика противоречива, то твой первокласнный математик окажется первокласным специалистом по бесполезному дерьму.

Кроме того чётко определить исходные понятия и способы вывода в необходимом и достаточном наборе должен любой математик, так как додумывать свои мысли честно до конца должно быть главным свойством математика. А то будет полный бред: исходные понятия хер пойми какие, какие хочу лоические средства, такими и херачу.
Аноним 27/03/15 Птн 09:40:37 #253 №251280 
>>251272

И да, в чем тут "комплекс неполноценности"?
Похоже у тебя не просто с логикой, а ты и смысла слов не понимаешь. У тебя походу асемасия.
Аноним 27/03/15 Птн 11:34:35 #254 №251324 
>>251272
> Можно ничего не знать о баллистике, но при этом всё равно кидать мяч по параболе.
Согласен.
Аноним 27/03/15 Птн 11:44:32 #255 №251327 
>>251279
> Если выяснится, что математика противоречива, то твой первокласнный математик окажется первокласным специалистом по бесполезному дерьму.
Если выяснится, что твои представления об устройстве прямой кишки противоречивы, ты срать не сможешь?
Аноним 27/03/15 Птн 11:47:39 #256 №251328 
>>251327
Если бы кишка работала в зависимости о моих знаниях о ней - да, не смог бы.
Аноним 27/03/15 Птн 11:55:48 #257 №251332 
>>251328
Ну так и твоя способность доказывать теоремы так же не зависит от знания логики.

>>251279
> Если выяснится, что математика противоречива, то твой первокласнный математик окажется первокласным специалистом по бесполезному дерьму.
Это ваще пушка. По-твоему, все результаты, полученные до 20 века можно выбросить, потому что тогда не было аксиом теории множеств?
Аноним 27/03/15 Птн 11:58:40 #258 №251335 
>>251332
> Ну так и твоя способность доказывать теоремы так же не зависит от знания логики.
Зависит исключительно от неё. Даже если я не знаю, что эти знания кто-то называет логикой.
> Это ваще пушка. По-твоему, все результаты, полученные до 20 века можно выбросить, потому что тогда не было аксиом теории множеств?
Ты думаешь, что цифры 2, 4, 6 и т.д. можно выбросить, потому что они были придуманы тогда, когда ничего не знали о чётных числах?
Аноним 27/03/15 Птн 12:22:57 #259 №251369 
>>251335
Ну так о четных числах и не нужно читать целую книгу, как ты предлагаешь поступить с логикой.
Аноним 27/03/15 Птн 12:26:07 #260 №251371 
>>251369
О логике тоже не нужно читать целую книгу. Логика - это формализация уже существующего механизма мышления в голове мартышки. Логичными были мыслители жившие и до рождества христова, и до аристотелева.
Аноним 27/03/15 Птн 12:27:09 #261 №251372 
>>251279
>Если выяснится, что математика противоречива, то твой первокласнный математик окажется первокласным специалистом по бесполезному дерьму.
А если рак на горе свистнет... Математика не противоречива. Противоречивой может оказаться какая то логическая система, но это не то же самое, что математика.
>>251335
В работе математика творчество важнее логики, или, по крайней мере, столь же значимо. Разумеется, рассуждениям придаётся точный смысл, но построение иерархии вплоть до самых низов никого не ебёт.
Аноним 27/03/15 Птн 12:31:18 #262 №251377 
>>251371
То есть ты сам признаешь, что знание определений из логики не дает тебе никакого преимущества по сравнению с людьми, жившими 2000 лет назад?
Аноним 27/03/15 Птн 13:07:38 #263 №251397 
>>251377
Нет, не признаю.
Аноним 27/03/15 Птн 13:39:38 #264 №251400 
>>251397
Это значит, что ты пидор
Аноним 27/03/15 Птн 13:46:27 #265 №251403 
>>251400
Нет, просто я вижу пользу логики, а ты её не понимаешь.
Аноним 27/03/15 Птн 13:46:40 #266 №251404 
>>251371
>Логика - это формализация уже существующего механизма мышления в голове мартышки
Вот оно - поколение взращенное научпопом.
sageАноним 27/03/15 Птн 14:05:02 #267 №251409 
>>251404
Твоё мнение вряд ли является истинным.
Аноним 27/03/15 Птн 14:25:02 #268 №251414 
>>251377
Владение логикой позволит тебе обосновать свою точку зрения другим мартыхам, и быть при этом понятым. По-моему, всё.
>>251404
А когда докинзоёбы буйствуют, ты где прячешься? Всё правильно, логика это маркетинг мысли. Если мысли нет - продавать нечего.
Аноним 27/03/15 Птн 14:59:51 #269 №251421 
>>251403
>вижу
рассказывай, что ты видишь. Мне логика ни разу в жизни не пригодилась, даже при доказательстве теорем.
Аноним 27/03/15 Птн 15:43:21 #270 №251426 
>>251332

> Ну так и твоя способность доказывать теоремы так же не зависит от знания логики.

Лол, как раз-таки без логики и произошёл жидкий обсёр, после которого все ринулись уточнять язык и исходные понятия. Луркай кризисы математики.

> Это ваще пушка. По-твоему, все результаты, полученные до 20 века можно выбросить, потому что тогда не было аксиом теории множеств?

Это из другой оперы, это психология и ограниченность мозга человеков не позволяющие оперировать формальными доказательствами большого размера, поэтому приходится читерить. Но это не означает, что уже найденному доказательству не предъявляются требования строгости.

>>251372

>А если рак на горе свистнет... Математика не противоречива. Противоречивой может оказаться какая то логическая система, но это не то же самое, что математика.

Он уже свистел и не раз, я писал об этом выше. Наивная математика противоречива, это доказано. Плюс не забываем, что упорядочивание и четкое выделение неопределяемых понятий это внутреннее свойство математики. Его нужно всегда проводить иначе будет бардак.

> В работе математика творчество важнее логики, или, по крайней мере, столь же значимо. Разумеется, рассуждениям придаётся точный смысл, но построение иерархии вплоть до самых низов никого не ебёт.

У тебя в голое каша. Одно другому не мешает, это вообще ортогональные вещи. Ты постоянно путаешь строгость математики как науки, с техническими способами придумать доказательство. Последнее вообще к делу не относится. Просто основы уже разработаны, и правильные методы доказательства тебе дадены. Ты ими пользуешься как потребитель, и то что ты не задумываешься или не хочешь думать об основах это только твоя личная проблема.

>>251421

Это потому, что нихуя ты в своей жизни не доказал.
Аноним 27/03/15 Птн 15:47:20 #271 №251427 
>>251327

Аналогия говно. Лучше представь врача который собирается удалять тебе рак прямой кишки имея фундаментально неправильное представление о ней.
Аноним 27/03/15 Птн 16:42:47 #272 №251454 
14274637670280.png
Анон, как решить эту систему в маткаде?

Второй день не хожу в универ, играюсь с дифференциальными уравнениями, строю траектории движения тел в силовых полях.

Сейчас три часа сидел с этой штукой, но так и не придумал, как скормить её ркфикседу или ещё чему-нибудь.
Аноним 27/03/15 Птн 17:02:20 #273 №251457 
>>251454
Перепиши dy/dx как (dy/dt)/(dx/dt), получишь y'/x'~f(x,y,t) . Комбинируя со вторым уравнением получишь систему дифуров в человеческом виде x'~f1(x,y,t), y'~f2(x,y,t).
Аноним 27/03/15 Птн 17:21:11 #274 №251459 
>>251457
Выразить в виде y'~f2(x,y,t) у меня не получается:

y'(t) / (1-[y'(t)]^2)^0.5 = (t+1-y)/(x-2)

y' имеет квадрат, да еще под корнем спрятан. Его обязательно оттуда выковыривать и явно выражать? Неужели больше никак?
Аноним 27/03/15 Птн 17:24:06 #275 №251460 
>>251457
или можно вот так?

x'~f1(x,y',y,t), y'~f2(x,x',y,t)
Аноним 27/03/15 Птн 17:40:58 #276 №251462 
>>251272
Ничего ты не знаешь о логике, например. И о науке, соответственно. Логика не формализация механизма, а именно инструмент для создания механизма для техничного построения обосновнной причинно-следственной связи, которая используется в рассуждении для получения верного вывода. Только она в рассуждениях используется, больше ничего. Математика никогда не противоречит логике, т.к математика и логика абсолютно неразрывные вещи. Математика давно включила в себя все, что даёт логика и самостоятельно разрабатывает новые методы и способы исходя из неопровержимых и очевидных логических аксиом по типу A=A. Логика как предмет - абсолютно бесполезная хуиня, которая ничего не может предложить для развития рассуждений. Аристотелевская логика и прочие логики в полном объеме сейчас нужны лишь для того, что бы дать логике точное определение. В то время, когда весь арсенал человеческого анализа и рассуждений имеется в ЕДИНСТВЕННОЙ науке - математике. Если же говорить за органическое определение логики как способности нашего интеллекта, то все очень неоднозначно. Это тоже самое, что дать определение сознанию человека.
Аноним 27/03/15 Птн 17:42:43 #277 №251463 
>>251426
>Лол, как раз-таки без логики и произошёл жидкий обсёр, после которого все ринулись уточнять язык и исходные понятия.
Этот «обсер» произошёл не из-за отсутствия логиков, а из-за их присутствия. Математики просто уточнили бы понимание и пошли дальше, но нет, надо развести истерию грандиозных масштабов на пустом месте, писать противоречивые аксиомы и самим же ужасаться от их противоречивости и т.д. и т.п.
>Наивная математика противоречива, это доказано
Очень хотелось бы узнать, что такое «наивная математика».
>неопределяемых понятий
Их нет. Если понятие неопределяемо под ним можно понимать что угодно и на его основе строить ничего нельзя. Какое-то определение есть всегда.
>Плюс не забываем, что упорядочивание и четкое выделение неопределяемых понятий это внутреннее свойство математики.
Что такое «внутреннее свойство математики»?
>У тебя в голое каша. Одно другому не мешает, это вообще ортогональные вещи. Ты постоянно путаешь строгость математики как науки, с техническими способами придумать доказательство. Последнее вообще к делу не относится. Просто основы уже разработаны, и правильные методы доказательства тебе дадены. Ты ими пользуешься как потребитель, и то что ты не задумываешься или не хочешь думать об основах это только твоя личная проблема.
То есть методы доказательства созданы логиками, а остальные просто пользуются готовым, решая чисто техническую задачу приложения глубоких идей логики к конкретной теоремке? При этом, как правило, даже не осознавая такого великого дара? Одна история охуительнее другой.
Я утверждаю не то, что математическая логика не нужна, а математические логики занимаются хуйнёй, лишь то, что реальная делятельность математических логиков с реальной деятельностью математиков имеет мало общего.
Аноним 27/03/15 Птн 17:44:07 #278 №251464 
>>251463

> Их нет. Если понятие неопределяемо под ним можно понимать что угодно и на его основе строить ничего нельзя. Какое-то определение есть всегда.

Тяжёлый случай. Ну давай определение символа, например.
Аноним 27/03/15 Птн 17:45:28 #279 №251466 
>>251459
Вырази x' =y' (x-2)/(y-t) из первого и подставь во второе. Получишь y'²=g(x,y,t). Тады x'²=1-g(x,y,t). С корнем в натуре надо будет разбирацца особо.
Аноним 27/03/15 Птн 17:45:30 #280 №251467 
>>251463

> Очень хотелось бы узнать, что такое «наивная математика».

Очевидно - математика использующая наивную теорию множеств в которой формулируются известные парадоксы.
Аноним 27/03/15 Птн 17:47:51 #281 №251468 
>>251467
Какие именно парадоксы в математике ты знаешь, например?
Аноним 27/03/15 Птн 17:52:31 #282 №251470 
>>251466
Спасибо. Что-то я стормозил.
Аноним 27/03/15 Птн 17:53:12 #283 №251472 
>>251462
>Логика как предмет - абсолютно бесполезная хуиня, которая ничего не может предложить для развития рассуждений.
Речь идёт именно о логике как о предмете. Остальная часть текста представляет собой какой-то невнятный поток сознания.
Аноним 27/03/15 Птн 17:54:19 #284 №251473 
>>251468

Парадокс дяди Рассела, бухали ФОРТИ.
Аноним 27/03/15 Птн 18:05:23 #285 №251474 
>>251464
Тут разные смыслы слова определение — в смысле построения системы, где, если его дать, придётся давать определения и всех составляющих слов и так никуда не придёшь, и в общем смысле — такое определение можно найти в любом словаре и оно нужно для того, чтобы пояснить, о чём идёт речь.
Аноним 27/03/15 Птн 18:11:24 #286 №251475 
>>251467
Математики до создания «наивной теории множеств» прекрасно без неё обходились, это показывает, что её использование не является необходимым, а самая интересная и, соответственно, используемая в математике часть теории множеств (понятие разных бесконечных мощностей, общая «философия»...) осталась неизменной и парадоксами не затронута.
>>251473
Напомнить, что по поводу парадокса Бурали-Форти говорил Кантор?
Аноним 27/03/15 Птн 18:23:55 #287 №251478 
>>251473
И какой это парадокс? Это не парадокс, а некорректность определения. Т.е. этот "парадокс" просто является доказательством того, что не существует так определённого объекта (множества всех множеств).
Аноним 27/03/15 Птн 18:28:43 #288 №251481 
>>251474

Поэтому они называют исходными понятиями и их невозможно определить. Зачем ты пытаешься исказить общепринятую терминологию и пишешь херню?

>>251475

>> Математики до создания «наивной теории множеств» прекрасно без неё обходились, это показывает, что её использование не является необходимым, а самая интересная и, соответственно, используемая в математике часть теории множеств (понятие разных бесконечных мощностей, общая «философия»...) осталась неизменной и парадоксами не затронута.

Там и других парадоксов было море, так как неопределяемых понятий было больше. Теория множеств сильно уменьшила их число.

Если само понятие противоречиво (это доказано), уже не имеет значения насколько интересны некоторые его следствия.

>> Напомнить, что по поводу парадокса Бурали-Форти говорил Кантор?

Он много чего говорил. Но это не имеет значения, если сравнить высказывания по философии математики математиков прошлого можно найти дофига взаимоисключающих параграфов.
Аноним 27/03/15 Птн 18:31:19 #289 №251482 
>>251478

В рамках наивной теории множеств это парадокс. Это уже во всяких аксиоматизациях пытались запретить себе формулировать определения таких множеств.
Аноним 27/03/15 Птн 18:39:41 #290 №251483 
>>251481
>Если само понятие противоречиво (это доказано), уже не имеет значения насколько интересны некоторые его следствия.
Имеет, возможно, понятия можно заменить на непротиворечивые, сохранив следствия.
>Он много чего говорил. Но это не имеет значения, если сравнить высказывания по философии математики математиков прошлого можно найти дофига взаимоисключающих параграфов.
Как будто сейчас лучше.
>>251482
Кантор такого понятия не формулировал, кстати. То есть он не считал, что противоречивые совокупности являются множествами и называл их «поистине бесконечными», вроде.
Аноним 27/03/15 Птн 18:43:10 #291 №251484 
14274709901070.png
>>251466
Вот.
Это была задача преследования. А корни при таких начальных условий будут просто положительные.

Но что делать с корнями, когда я запилю задачу с несколькими телами, которые будут преследовать друг друга?
Аноним 27/03/15 Птн 20:19:00 #292 №251507 
Я помню, у нас в универе философию вел дед, который любил спрашивать хуйню наподобие "а что такое время?" и потом говорил: "вот, в курсе философии мы определили это понятие и теперь вы можете им пользоваться". Вот этот даун, который форсит логику, такой же отбитый.
Аноним 27/03/15 Птн 20:36:12 #293 №251509 
>>251481
>Если само понятие противоречиво (это доказано), уже не имеет значения насколько интересны некоторые его следствия.
Только при очень поверхностном взгляде. Можно или уточнить понятие, или воспользоваться неклассической логикой, или ещё что-нибудь сделать. Можно просто отбросить противоречивое понятие, принять одно из следствий в качестве отправного и вывести остальные интересные следствия из него.
Аноним 27/03/15 Птн 20:38:21 #294 №251510 
14274779012910.png
14274779012981.png
Читаю "Курс алгебры" Винберга. Решил задачу 2 на пикрелейтед брутфорсом, и у меня такое ощущение, что ее можно как-то красиво решить. Есть идеи у кого-то? Мое решение:
1) Пусть O1 - точка пересечения ax и by.
Решаю систему
O1 = kb + (1 - k)y
O1 = ha + (1 - h)x
относительно k и h. Получаю O1 = const (lambda mu, 1, lambda)
2) Аналогичным образом нахожу барицентрические координаты остальных точек пересечения.
3) Применяю к ним теорему 2 и получаю, что lambda mu nu = 1
Аноним 27/03/15 Птн 20:44:28 #295 №251512 
>>251509

1) Это будет уже другое, более ограниченное понятие
2) При этом ты будешь широко использовать матлогику и её методы, которую ты хочешь выкинуть
Аноним 27/03/15 Птн 20:47:46 #296 №251514 
>>251463

> Что такое «внутреннее свойство математики»?

Это я пытался выкрутиться чтобы не писать "внутренняя логика математики", так как слово логика уже занята в другом смысле в той же фразе. Имеется ввиду, что математика стремится к строгости, поэтому естественно как продолжение это строгости разложить все базовые понятия и средства доказательства теорем максимально точно.
Аноним 27/03/15 Птн 20:55:39 #297 №251515 
>>251463

> То есть методы доказательства созданы логиками, а остальные просто пользуются готовым, решая чисто техническую задачу приложения глубоких идей логики к конкретной теоремке?

Именно так. Только основания на 95% разработали все же математики или спецы по матлогике. И фразу ты криво построил. Обычный математик пользуется не методами и идеями мат логики, а исходным понятийным аппаратом и допустимыми средствами доказательства которые "утвердила" мат логика.

> реальная делятельность математических логиков с реальной деятельностью математиков имеет мало общего.

В который раз тебе повторить, что никто не заставляет мыслить и искать доказательство исключительно внутри формальной системы? Ты путаешь поиск доказательства и последующее его строгое оформление, такое что теоретически оно может быть выполнено в подходящей формальной системе.
Аноним 27/03/15 Птн 20:59:50 #298 №251516 
>>251515
Никто не оформляет свои доказательства в какой-то формальной системе, за исключением крошечной горстки фриков от логики.
http://arxiv.org/archive/math - можешь убедиться.
Аноним 27/03/15 Птн 21:06:52 #299 №251517 
>>251514
Математика, безусловно, стремиться к строгости, точнее, точности, но есть небольшая разница между математическим и мат. логическим подходом. Математик хочет найти наиболее общие способы рассуждения, а логик — наиболее элементарные, базовые. По-моему пора прекращать этот спор.
Аноним 27/03/15 Птн 21:14:00 #300 №251519 
Лол
http://arxiv.org/pdf/1503.07848.pdf
Аноним 27/03/15 Птн 21:14:31 #301 №251520 
>>251519
Он еще и в ворде сверстал
Аноним 27/03/15 Птн 21:27:19 #302 №251523 
>>251519
А в чём лулз?
Аноним 27/03/15 Птн 21:39:43 #303 №251525 
>>251523
В контрасте с любой другой статьей оттуда.
Аноним 27/03/15 Птн 22:19:27 #304 №251532 
14274839678550.png
Анон, выручай снова.

Я сформировал таблицу rkfixed, там 12 колонок. Хочу вывести на график u^2 и u^5 (или любые другие колонки), а он мне в ответ пишет вот это (см. пик).

Что я делаю не так? Как мне визуализировать это всё?
Аноним 27/03/15 Птн 22:39:04 #305 №251535 
>>251516

Я не говорил, что пишут доказательства в формальной системе, я писал о принципиальной возможности это сделать. Никто же не оперирует бесконечно малыми как при Лейбнице.

>>251517

Ни разу не согласен, и тот и другой ищут наиболее общие и базовые понятия и рассуждения. В этом суть математики.

Аноним 28/03/15 Суб 00:05:33 #306 №251546 
Матаны, а вот поясните, правильно ли я понимаю теоремы Геделя. Правильно ли, что любая формальная система неполна? Правильно ли что, чтобы доказать непротиворечивость любой четко формализованной системы нужно обязательно выйти за рамки этой системы. Т.е. построить другую формализованную теорию, и в рамках неё доказать первую. Для доказательства непротиворечивости второй теории строим третью и так далее до бесконечности? И о чем это нам говорит? Что хуета вся эта ваша математика, в ней невозможно построить так называемую непротиворечивую "Теории Всего". А раз это невозможно сделать даже в абстрактной науке, то что вообще можно говорить про физику? Мы никогда не докопаемся до СУТИ. Бог всегда будет ускользать от нас за предел.
Аноним 28/03/15 Суб 00:07:37 #307 №251547 
Парни, что есть годного по хаосу и фракталам? Очень бегло (в среднем по 20 сек) просмотрел базовые книги, вот к чему я пришел:
1) Мандельброт Б. Фрактальная геометрия природы.
Мешанина из охуительных историй об истории научной работы самого Мандельброта и научно-популярной составляющей, мол "как нам обустроить Россию и посчитать береговую линию Британии?", то есть приложение теории хаоса к физике, химии и т.д. на уровне школы.
2) Шредер. Фракталы, хаос, степенные законы.
То же, что и предыдущая книга, может даже более популярно.
3) Кроновер. Фракталы и хаос в динамических системах.
Как мне показалось, это книга о том, как строить фракталы и только. Малый объем, большой шрифт, даже строчки программного кода приводит, мол "введи и посмотри, парень". Не совсем ясно, какова ценность этого учебника именно как учебника.
Пара англоязычных книг:
1) James Gleick. Chaos: The Making of a New Science.
Насколько я понял, это полностью научно-популярное чтиво, Нью-Йорк Таймс Бестселлер, на первых 200-ах страницах которого нет фактически никаких формул, одна только история и картиночки фракталов.
2) Albert-László Barabási. Fractal concepts in surface growth.
Серьезные щи и годная книга, но, насколько я понял, ускоспециализированная, для людей с базой.
Аноним 28/03/15 Суб 00:13:58 #308 №251548 
>>251546
Неправильно. Гёдель сформулировал интересующую тебя теорему лишь для некоторого, довольно узкого, класса формальных арифметик. Обычная же логика вполне себе полна. То есть существуют формальные системы, полнота которых доказана, исчисление высказываний например. Полноту исчисления предикатов доказал тот же самый Гёдель, это так называемая теорема Гёделя о полноте.
https://ru.wikipedia.org/wiki/Теорема_Гёделя_о_полноте
Аноним 28/03/15 Суб 00:16:36 #309 №251549 
>>251547
>Парни, что есть годного по хаосу и фракталам?
Ну ты и темы выбрал.
Аноним 28/03/15 Суб 00:16:53 #310 №251550 
>>251535
Это просто означает, что сама идея формальной системы недостаточна для построения оснований математики, или, по крайней мере, неудобна. Теоретикам оснований следует перестать обсасывать формальные системы и придумать что-нибудь более новое.

Алсо, актуальные бесконечно-малые есть в нестандартном анализе.
Аноним 28/03/15 Суб 00:20:18 #311 №251552 
>>251549
А что такого?
Аноним 28/03/15 Суб 00:27:42 #312 №251557 
>>251548
Ну так ведь то всего лишь о предикатах, т.е. об отдельных утверждениях или об отдельных функциях. Ведь на сегодняшний день не существует непротеворечивых теорий с доказанной полнотой?
>Гёдель сформулировал интересующую тебя теорему лишь для некоторого, довольно узкого, класса формальных арифметик.
Почему же узкого? Обобщенная формулировка говорит:
Всякая достаточно сильная рекурсивно аксиоматизируемая непротиворечивая теория первого порядка неполна.
Аноним 28/03/15 Суб 00:33:48 #313 №251559 
>>251557
Существуют, конечно же. Исчисление высказываний непротиворечиво и полно. Исчисление предикатов первого порядка непротиворечиво и полно.

Слова "достаточно сильная" означают, что среди аксиом формальной теории содержатся аксиомы формальной арифметики. Что это такое, можешь прочитать в книге Клини.
Аноним 28/03/15 Суб 00:39:20 #314 №251560 
>>251559
Т.е. согласно Геделю, неполны только те теории, которые опираются на аксиомы Пеано?
https://ru.wikipedia.org/wiki/Аксиомы_Пеано
Аноним 28/03/15 Суб 00:46:31 #315 №251563 
>>251560
Кроме аксиом Пеано, существуют другие аксиомы. Но суть такая, да.
Аноним 28/03/15 Суб 00:55:45 #316 №251567 
>>249033
Матаны, поясните по хардкору за фракталы.
Посмотрел https://vimeo.com/103303696 и решил вырастить цифровое дерево, но в математику никогда не мог.
Беру blender3d и питон, но какими формулами обмазаться?
sageАноним 28/03/15 Суб 02:59:18 #317 №251587 
>>251563
Не вводи людей в заблуждение. Теорема Гёделя верна для любой формальной системы, т.к. доказана для теории чисел, самой обобщённой из всех (любой формализм так или иначе опирается на неё).
https://2ch.hk/ph/res/47340.html#47353
Аноним 28/03/15 Суб 02:59:50 #318 №251588 
>>251546
-- >>251587
Аноним 28/03/15 Суб 03:00:19 #319 №251589 
>>251547
Пригожин.
sageАноним 28/03/15 Суб 03:01:51 #320 №251590 
>>251550
> Теоретикам оснований следует перестать обсасывать формальные системы и придумать что-нибудь более новое.
И пока так получается, что что бы они не выдумали, это всё сводится к формальной системе.
Аноним 28/03/15 Суб 03:25:54 #321 №251596 
14275023544600.jpg
>>251590
Место проклятое.
sageАноним 28/03/15 Суб 03:30:03 #322 №251597 
>>251596
Нет, просто формализм "формальная система" пока достаточно общ.
Аноним 28/03/15 Суб 03:32:17 #323 №251598 
>>251587
Все правильно тот анон сказал, уебывай в свою "философию" обратно.
sageАноним 28/03/15 Суб 03:33:20 #324 №251599 
>>251598
> Мне нравится то говно, не мешай его жевать.
Хорошо.
Аноним 28/03/15 Суб 03:36:30 #325 №251600 
>>251599
4 часа утра, я слишком устал и хочу спать, чтобы тебе фелосафу что-либо пояснять детально. Завтра.
sageАноним 28/03/15 Суб 03:36:37 #326 №251601 
Рекомендую научпоп-книгу "Гёдель, Эшер, Бах", написанную математиком Дугласом Хофштадтером как раз про тот философский смысл, о котором говорит философ-кун.
sageАноним 28/03/15 Суб 03:37:30 #327 №251602 
>>251600
Пояснения для тебя уже оформлены в книгу для самых маленьких: >>251601 .
Аноним 28/03/15 Суб 03:41:34 #328 №251603 
>>251587
>Теорема Гёделя верна для любой формальной системы
Вздор. Не знаешь - не говори, ну.
sageАноним 28/03/15 Суб 03:42:24 #329 №251604 
>>251603
> т.к. доказана для теории чисел, самой обобщённой из всех (любой формализм так или иначе опирается на неё).
sageАноним 28/03/15 Суб 03:51:15 #330 №251605 
Парадокс, кстати, и в том, что сама эта теорема сформулирована в терминах теории чисел, формальной системы. Т.е., истинность математики формально недоказуема, и чудо сознательного бытия в том, что пока она "совпадает" с поведением физической реальности. Доказательством математики является существование разума вообще, и в этой фундаментальной проблеме формализации процесса мышления математика она пересекается с проблемой наблюдателя в физике - того, кто "следит", чтобы реальность соблюдала физические законы.
sageАноним 28/03/15 Суб 04:00:34 #331 №251606 
>>251605
Cogito ergo sum. "Я мыслю, значит я существую" говорит наблюдатель из физики. "Я мыслю, значит свойства физической реальности сводимы к математике" как бы отвечает ему вычислитель из математики.
sageАноним 28/03/15 Суб 04:17:59 #332 №251610 
>>251606
До Декарта примерно о том же и Платон размышлял, разделив модель реальности (и методы её познания) на мир идей (по-сути - математику, логический вывод) и мир вещей (по-сути - физику, эмпирический опыт).
Аноним 28/03/15 Суб 04:18:00 #333 №251611 
14275054804660.png
>>251604
Ну что ты несёшь, а? Вот теорема. Хватит спорить.
Сорс - Клини, "Введение в метаматематику".
sageАноним 28/03/15 Суб 04:19:28 #334 №251612 
>>251611
Не очень понял, как твой пик опровергает сказанное мною.
Аноним 28/03/15 Суб 04:21:25 #335 №251613 
>>251612
>Теорема Гёделя верна для любой формальной системы
> > любой
>только для системы из гл. IV
sageАноним 28/03/15 Суб 04:22:56 #336 №251615 
>>251613
> все формальные системы сводимы к формализму из гл. IV.
Аноним 28/03/15 Суб 04:25:26 #337 №251616 
14275059265270.png
>>251615
Вздор. Вот картинка.
Если принять аксиомы A1, получится полная и непротиворечивая формальная система.
Если принять аксиомы A1 и A2, получится полная и непротиворечивая формальная система.
Но вот если принять и A1, и A2, и B, то теорема Гёделя будет справедлива.
sageАноним 28/03/15 Суб 04:28:20 #338 №251617 
>>251613
> получится полная и непротиворечивая формальная система.
Но вот формально доказать непротиворечивость и полноту твоей системы в терминах самой этой системы у тебя не получится - и теорема Гёделя только об этом.
Аноним 28/03/15 Суб 04:30:27 #339 №251618 
>>251617
Не получится только для системы A1,A2,B, и только для неё справедлива теорема Гёделя. Для других систем эта теорема не справедлива. Для системы A1 очевидно получится, это доказывается простой проверкой. Теорема о полноте исчисления высказываний - общеизвестная теорема.
sageАноним 28/03/15 Суб 04:33:20 #340 №251619 
>>251618
Доказать можно либо полноту, либо истинность, но не одновременно. Для любой формальной системы, которая способна формулировать свою собственную полноту или непротиворечивость.
sageАноним 28/03/15 Суб 04:34:21 #341 №251620 
>>251618
> очевидно
Но, увы, не формально.
Аноним 28/03/15 Суб 04:34:23 #342 №251621 
>>251619
Я прав, а ты нет.
Аноним 28/03/15 Суб 04:35:02 #343 №251622 
>>251620
Просто прочитай уже учебник.
sageАноним 28/03/15 Суб 04:40:09 #344 №251623 
>>251621
Теорема Гёделя, похоже, хороший "водораздел" между математиками и вычислителями дифуров под водочку.

Ты можешь доказать и истинность, и полноту некой формальной системы только в терминах другой формальной системы, включающей рассматриваемую. Для того, чтобы доказать истинность своего предыдущего доказательства, тебе придётся надстроить следующую формальную систему, включающую в себя ту, в которой ты доказывал истинность рассматриваемой, и т.д. до бесконечности. В этом суть гёделевской теоремы.
sageАноним 28/03/15 Суб 04:40:45 #345 №251624 
>>251622
Посоветую сделать тебе то же самое.
Аноним 28/03/15 Суб 04:45:59 #346 №251625 
>>251623
>истинность
Речь о непротиворечивости. Не смешивай термины. С примером, опровергающим твоё заявление, ты можешь ознакомиться в любом нормальном учебнике логики. Например, в книжке Клини.
sageАноним 28/03/15 Суб 04:49:43 #347 №251626 
>>251625
> Речь о непротиворечивости. Не смешивай термины.
Истинностью в математике является непротиворечивость.
> С примером, опровергающим твоё заявление, ты можешь ознакомиться в любом нормальном учебнике логики.
Ты можешь доказать и истинность, и полноту некой формальной системы только в терминах другой формальной системы.
Аноним 28/03/15 Суб 04:52:55 #348 №251627 
>>251626
Нет, ты не прав. Почему ты так упорно несёшь чушь?
sageАноним 28/03/15 Суб 04:54:00 #349 №251628 
>>251626
> Ты можешь доказать и истинность, и полноту некой формальной системы только в терминах другой формальной системы, которая, в свою очередь, и сама нуждается в посторонней системе, чтобы доказать свою истинность и полноту.
minifix
sageАноним 28/03/15 Суб 04:54:30 #350 №251629 
>>251627
Это не чушь, а математика.
Аноним 28/03/15 Суб 04:55:43 #351 №251630 
>>251629
Нет. Это выдуманная тобой чушь.
sageАноним 28/03/15 Суб 05:01:49 #352 №251631 
В стандартной интерпретации гёделева неразрешимая формула A означает «не существует вывода формулы A», то есть утверждает свою собственную невыводимость в системе S. Таким образом, A является аналогом парадокса лжеца. Рассуждения Гёделя в целом очень похожи на парадокс Ришара. Более того, для доказательства существования невыводимых утверждений может быть использован любой семантический парадокс.
Аноним 28/03/15 Суб 05:05:00 #353 №251632 
>>251631
Существуют формальные системы, для которых сабжевая теорема Гёделя неверна. Например, исчисление высказываний. Ты это признаёшь?
sageАноним 28/03/15 Суб 05:11:37 #354 №251633 
14275086975160.jpg
Отрицание глобальности теоремы Гёделя (вплоть до недоказуемости истинности/непротиворечивости самой математики) схоже с отрицанием ОТО адептами ньютоновской физики. И те, и другие не могут помыслить условия, при которых их упрощённое понимание перестаёт быть корректным. Сложность пугает.
Аноним 28/03/15 Суб 05:13:33 #355 №251634 
14275088138700.jpg
>>251633
sageАноним 28/03/15 Суб 05:16:21 #356 №251635 
>>251632
Она верна для любых формальных систем. В рамках исчисления высказываний тоже не получится доказать непротиворечивость и полноту исчисления высказываний, это можно сделать только с привлечением других формальных систем.
Аноним 28/03/15 Суб 05:23:39 #357 №251636 
14275094191800.jpg
>>251635
Итак, ты не знаешь, о чём говоришь. Ещё раз повторю: ты не прав, читай учебник.
sageАноним 28/03/15 Суб 05:28:33 #358 №251637 
>>251636
Нет ты.
Аноним 28/03/15 Суб 05:28:57 #359 №251638 
14275097375660.png
Сцы, сцы никогда не меняется.
Подскажите раздел с меньшим количеством разбивающих лоб фанатиков, что ли.
sageАноним 28/03/15 Суб 05:31:39 #360 №251640 
>>251638
Чтобы ты увеличил там количество фанатиков на единицу?
Аноним 28/03/15 Суб 05:32:04 #361 №251641 
>>251638
Да ладно, интересно же. Маленькие радости аутиста.
Аноним 28/03/15 Суб 05:37:46 #362 №251643 
>>251640
В чём же заключается мой фанатизм, няша?
sageАноним 28/03/15 Суб 05:38:59 #363 №251644 
>>251643
А сам как думаешь?
sageАноним 28/03/15 Суб 05:41:52 #364 №251645 
Про исчисление высказываний вынужден согласиться.
Исчисление высказываний является непротиворечивой, полной, разрешимой теорией, причем все три утверждения доказуемы в рамках самой логики высказываний.
Однако,
Несмотря на свою важность и широкую сферу применения, логика высказываний является простейшей логикой и имеет очень ограниченные средства для исследования суждений.
Т.е., математику на ней не построишь.
Аноним 28/03/15 Суб 05:52:57 #365 №251646 
>>251644
Либо ты меня с кем-то путаешь, либо манёвры включил.
sageАноним 28/03/15 Суб 05:57:10 #366 №251647 
В общем, я был бы формально прав, если бы сделал акцент на словах "достаточно сложная формальная система" (способная сформулировать арифметику), которые я сам для себя некорректно вынес за скобки, обобщив до "любая формальная система". Спасибо за новые знания (уточнение старых). Однако общий посыл о невозможности формального доказательства истинности+полноты гипотетической "теории всего" (как формальной системы, способной выразить как минимум арифметику) философ-куна остался верным.
sageАноним 28/03/15 Суб 05:59:43 #367 №251648 
>>251646
Да, это маневрирование. Хотел свести наш спор к твоему батхёрту.
Аноним 28/03/15 Суб 06:01:50 #368 №251649 
>>251648
Я с тобой ни о чём не спорил.
sageАноним 28/03/15 Суб 06:02:23 #369 №251650 
>>251649
Ну вот, опять.
sageАноним 28/03/15 Суб 06:05:23 #370 №251651 
Кароч, если ты мимокрок, то я не с тобой спорил, но принял за того, с кем спорил. Но ты такая кокетка, и не можешь позволить себе просто написать "я мимокрок", оставляя собеседника в гаданиях, где же оборвалась нить диалога.
Аноним 28/03/15 Суб 06:05:28 #371 №251652 
>>251645
>>251647
Угу. Хорошо, что ты умеешь уточнять свою позицию.
Аноним 28/03/15 Суб 06:06:26 #372 №251653 
>>251651
это аиб, детка
здесь только анон
sageАноним 28/03/15 Суб 06:08:50 #373 №251654 
>>251653
Не место красит человека.
Аноним 28/03/15 Суб 06:09:38 #374 №251655 
>>251654
А маляр.
sageАноним 28/03/15 Суб 08:16:17 #375 №251658 
Фу, блять, до Гёделя уже скатились пидарасы. Пруфают научпопом и тредами из ph/. Сажи.
Аноним 28/03/15 Суб 08:56:24 #376 №251664 
>>251658
Не пруфают, а цитируют философские выводы из этой теоремы, балбес.
sageАноним 28/03/15 Суб 09:19:21 #377 №251669 
>>251664
Нахуй пошёл, со своими философскими выводами.
sageАноним 28/03/15 Суб 09:36:50 #378 №251671 
>>251669
Ну чёты не хейть меня))))
Аноним 28/03/15 Суб 11:32:42 #379 №251682 
>>251631

А что значит существует?
Аноним 28/03/15 Суб 11:36:29 #380 №251683 
>>250014

БАмп вопросу.
Аноним 28/03/15 Суб 12:08:17 #381 №251691 
Гоните посаны этого фелосафа отсюда ссаными тряпками!
Аноним 28/03/15 Суб 12:19:09 #382 №251694 
>>251523
Это не понравилось бы вербиту.
Аноним 28/03/15 Суб 12:29:20 #383 №251696 
>>251691

Которого???
sageАноним 28/03/15 Суб 12:29:57 #384 №251697 
>>251691
Лучше уж философы, чем такие вот радикальные школьники-максималисты.
Аноним 28/03/15 Суб 12:32:10 #385 №251699 
>>251683
>>250014
Лучше бы уточнить реквест. Уровень знакомства с вопросом? Интересуют ли различных подходов к основаниям математики или только один, например, теоретико-множественный? Необходимо строгое изложение математики или же философское обсуждение?
Аноним 28/03/15 Суб 13:43:01 #386 №251723 
>>251426
>Это потому, что нихуя ты в своей жизни не доказал.
Как минимум все задачи-теоремы в Зориче. Или они не используют логику?
Аноним 28/03/15 Суб 13:59:01 #387 №251731 
>>251636
Ты не прав, читай оригинал.
Аноним 28/03/15 Суб 14:52:09 #388 №251739 
>>251731
Сам Гёдель в оригинальной статье доказал первую теорему о неполноте лишь для теории типов из Principia Mathematica.
мимо
Аноним 28/03/15 Суб 15:41:52 #389 №251747 
14275465125740.jpg
Подскажите, специалисты, какую-нибудь короткую книжку-обзор по дифгему и римановой геометрии ( а ещё хотелось бы по дифтопу, но это уже как максимум ) , для тех кому нужно вспомнить. Сам я хочу в алгебру, дифгем сдал года три назад и больше к нему не прикасался, но какие-то основные результаты, как я понимаю, знать надо каждому, а времени у меня не много.
Аноним 28/03/15 Суб 19:22:04 #390 №251787 
14275597248230.jpg
Даже хочется новый тред, т.к. этот зашкварен несостоятельным и неконструктивным недоспором про логику и математику.
Аноним 28/03/15 Суб 21:20:06 #391 №251809 
> Логика и метаматематика это все части математики, т.к. без них математики не существует.
Аноним 28/03/15 Суб 21:59:01 #392 №251818 
>>251510
Бамп вопросу. Аноны, неужели ни у кого нет идей? Тут же все дрочат на этот учебник.
Аноним 28/03/15 Суб 23:35:39 #393 №251843 
>>251747
Насчёт дифф. топологии - либо Хирш, либо Милнор,Уоллес. Обе книги называются "Дифференциальная топология"
Аноним 29/03/15 Вск 00:39:30 #394 №251849 
>>251787
> неконструктивным недоспором
> тред на сосаче
Аноним 29/03/15 Вск 01:13:50 #395 №251854 
Аноны, кто в НМУ обучается, расскажите как там.
Аноним 29/03/15 Вск 01:26:03 #396 №251855 
>>251818
>Тут же все дрочат на этот учебник.
О, у меня возник другой вопрос. В сети можно найти дохуище отличных сканов различных книг (издательств Лань, МЦНМО и т.д.), возникает такое чувство, что это инсайдеры выкладывают исходники. Но там, блять, дохуище ошибок. Наверняка всем известно огромное количество ошибок в гуляющей по сети пдф-ке "Что такое математика?" Куранта. Хотя мне недавно удалось повертеть саму книгу в руках и замеченных ошибок я не увидел. Сейчас читаю "Дискретную математику для инженера" Кузнецова (издание 2009 года), когда чувствую, что что-то тут не так, лезу в издание 1988 года и обычно нахожу, что мои подозрения - справедливы.
Вопрос: это такие дауны-редакторы/корректоры сейчас сидят в издательствах научной литературы или они вбрасывают в сеть пдф-ки с ошибками?
Аноним 29/03/15 Вск 02:09:36 #397 №251856 
>>249773
ну хз тогда, я думал это Ацюковский.
Аноним 29/03/15 Вск 02:25:06 #398 №251859 
14275851063340.png
Почему функция D не может быть использована тут?
Аноним 29/03/15 Вск 03:18:21 #399 №251870 
>>251856
В общем-то, это Жан-Пьер Серр.
Аноним 29/03/15 Вск 03:21:05 #400 №251871 
>>251870
нюфани совсем уже зажрались, Серра не знают
Аноним 29/03/15 Вск 11:37:10 #401 №251916 
>>249033
кто сегодня, помимо Мочидзуки, пишет фундаментальные статьи в математике (ну типа математиков 50ых-60ых)?
Аноним 29/03/15 Вск 12:39:55 #402 №251935 
>>251916
http://arxiv.org/archive/math
Аноним 29/03/15 Вск 15:02:09 #403 №251961 
>>251855
Я думаю это проблема русских учебников. У авторов нет мотивации исправлять ошибки и выдрачивать каждую букву в учебнике, потому что в рашке никто не купит книгу, если ее можно украсть в интернете. А когда книга стоит 100 баксов и покупатели есть, автор и ошибки исправляет, и solution manual издает.
Аноним 29/03/15 Вск 16:46:47 #404 №252017 
>>251961
>У авторов нет мотивации исправлять ошибки
Ну так суть в том, что в советском оригинале таких ошибок не было.
Аноним 29/03/15 Вск 18:10:49 #405 №252050 
>>251916
Вербицкий.
Аноним 29/03/15 Вск 18:26:12 #406 №252062 
14276427728950.png
14276427728961.png
Пацаны, хелп. Нужно доказать, что замыкание выпуклого множества выпукло. Почему последнее утверждение в моем доказательстве (p'q' проходит через окрестность x) верно? Вообще вы доказываете такие вещи? С одной стороны, это очевидно, с другой стороны, хочется доказать.
Аноним 29/03/15 Вск 18:33:57 #407 №252071 
>>252062
Забыл сказать, матан я топологию я не знаю.
Аноним 29/03/15 Вск 19:12:47 #408 №252102 
14276455672800.png
>>252062
Аноним 29/03/15 Вск 23:57:35 #409 №252251 
14276626555880.png
>>252062
>Почему последнее утверждение в моем доказательстве (p'q' проходит через окрестность x) верно?
А оно верно?
Аноним 30/03/15 Пнд 00:16:41 #410 №252260 
Уважаемые матаны. Пишет вам анон и соседнего https://2ch.hk/sci/res/212837.html итт треда. В ходе решения уравнения теплопроводности получил рис 1. Необходимо найти коэффициенты Cn. Подскажите, как это сделать?
Аноним 30/03/15 Пнд 00:17:56 #411 №252261 
14276638762940.png
>>252260
Рис. 1.
Аноним 30/03/15 Пнд 00:23:47 #412 №252265 
>>252261
Кривое какое-то выражение
Аноним 30/03/15 Пнд 01:03:39 #413 №252276 
>>252251
Радиус один и тот же.
Аноним 30/03/15 Пнд 01:14:50 #414 №252278 
>>252062
Нам надо соединить две точки замыкания отрезком, лежащим в множестве.
Берем последовательность отрезков, соединяющих точки исходного множества, найденные в e-окрестностях двух точек замыкания. Последовательность отрезков имеет предел в замыкании, это - отрезок, соединяющий нужные две точки.
Аноним 31/03/15 Втр 10:25:44 #415 №252305 
14277867446120.jpg
Откат?
Аноним 31/03/15 Втр 11:11:32 #416 №252307 
Что за откат, все посты про основания стёрлись.
Аноним 31/03/15 Втр 11:29:59 #417 №252309 
>>252307
заебись. Может и мудак с их невероятной важностью съебет.
Аноним 31/03/15 Втр 16:12:19 #418 №252361 
>>252307
> Что за откат, все про основания стёрлись.
Вы прослушали краткую лекцию по истории оснований математики.
Аноним 31/03/15 Втр 17:06:07 #419 №252372 
>>252361
>>252309
А я писал тот длинный пост с рекомендациями о том, что почитать по основаниям. Как-то немного обидно.
Аноним 31/03/15 Втр 17:09:39 #420 №252373 
>>252372
Тезисно перепиши, список тем, список книг.
Аноним 31/03/15 Втр 18:55:38 #421 №252397 
>>252373
Лень, конечно. Ну ладно, повторю основную часть списка.
Reverse Mathematics(!): введение к Simpson Subsystems of Second Order Arithmetic
Ordinal Analisis: M.Ratjen The art of ordinal analisis
Форсинг(!): сам метод в T.Jech Set Theory 2003(важно, нужно именно издание 2003) 13 глава и хорошо бы посмотреть на примеры результатов на его основе - погугли обзор или просмотри туже книгу Йеха.
Большие кардиналы: ?
Аксиома детерминированности: Recent progress on the continuum hypothesis + где-нибудь еще почитать, про особенности математики при замене AC на AD.
Бонус: H.M. Friedman Philosophical problems in Logic Lecture Notes http://u.osu.edu/friedman.8/files/2014/01/Princeton532-1pa84c4.pdf
Классические школы конструктивизма Маркова и Бишопа: посмотреть, как они строят конструктивный анализ и т.п. в какой-нибудь из книг соответствующей школы.
Интуиционистские теории типов:? (есть оригинальная статья Мартин-Лёфа ~1980, но её нельзя рекомендовать в качестве обзора)
Мелкие, но важные темы, касающиеся интуиционизма(!): BHK интерпретация, изоморфизм Карри-Говарда, "Dialectica" interpretation.
Неполнота: введение к H.M. Friedman Concrete Mathematical incompleteness https://u.osu.edu/friedman.8/files/2014/01/0.Intro061311-17n3hse.pdf.

Во всех случаях, кроме видимо конструктивизма, насколько я помню, англ. википедия в целом адекватна и её не бессмыслено посмотреть в обзорных целях до указанных источников или чтобы посмотреть какие есть источники. Также стоит обратить внимание на статьи в стэнфордской энциклопедии философии, касающиеся оснований.
Аноним 31/03/15 Втр 19:37:02 #422 №252404 
>основания
>интуиционизм
>конструктивизм

Снова сёмы говна накидали в тему. Абу, откатывай ещё раз.
Аноним 31/03/15 Втр 19:50:00 #423 №252405 
>конструктивизм
>Канторосказки
чет я уже ничего не понимаю
Аноним 31/03/15 Втр 20:16:49 #424 №252412 
>>252404
>>252405
Грустно, что уровень раздела столь низок, что, если пишешь что-то про основания, то в тебе видят фанатика, который начнет впихивать свои философские взгляды и заявлять, что никакой математики вне его взглядов на основания нет (ну или что-то на том же уровне адекватности).
Аноним 31/03/15 Втр 21:42:19 #425 №252434 
>>252412
>что никакой математики вне его взглядов на основания нет

Но именно так ты и заявляешь ну или не ты, а другой хуй, который тут всем ебёт мозги основаниями:
>> Логика и метаматематика это все части математики, т.к. без них математики не существует.
Аноним 31/03/15 Втр 22:14:57 #426 №252436 
>>252434
>ну или не ты,
Ровно об этом я говорил, когда почти прямым текстом назвал фанатиком автора вот этой глупости (с тем, что математическая логика часть математики я согласен, с остальным - нет)
>>> Логика и метаматематика это все части математики, т.к. без них математики не существует.
Мне она тоже запомнилась
Аноним 31/03/15 Втр 23:01:08 #427 №252446 
>>252436
А я не согласен. Математическая логика - это отдельная наука, которую разумно причислять скорее к классу информатики.
Аноним 31/03/15 Втр 23:28:17 #428 №252450 
>>252446
теория топосов тогда - это математика или логика?
Аноним 31/03/15 Втр 23:40:05 #429 №252453 
>>252446
Если посмотреть на предмет изучения мат.логики, её происхождение и принятые в ней методы исследований - это совершенно однозначно часть математики. Единственная потенциальная причина считать иначе, которую я вижу - это относительно слабая связь с другими разделами. Но объявление раздела не частью математики лишь на таком основание - это объявление, что предмет изучения математики стал уже, чем считалось ранее. Я подобную практику не одобряю, на мой взгляд - это приватизация названия лишь частью исходной области.
Аноним 31/03/15 Втр 23:51:17 #430 №252456 
14278350779340.png
>>252453
Предмет изучения математической логики скорее принадлежит информатике, нежели математике.
Математическую логику создал Лейбниц в своём трактате "Об искусстве комбинаторики". Этот трактат очевидно принадлежит информатике, пикрелейтед.
В математике не используются методы, которые применяются математической логикой. Математики крайне редко строят формальные модели, тогда как главный инструмент математической логики - это именно формальный вывод.
Математическая логика слабо связана с разделами математики, но весьма сильно связана с разделами информатики.
Аноним 01/04/15 Срд 00:43:38 #431 №252461 
по-моему матлогика - идеально вписывается в алгебру, так что норм её считать математикой, расслабтесь, йоу, получайте удовольствие просто от всего этого.
Вот то ли дело матстат.
Аноним 01/04/15 Срд 01:34:09 #432 №252463 
>>252456
Честно говоря слабо представляю, что там делал Лейбниц, но в любом случае мат.логика в сколь-нибудь современном виде появилась только во второй половине 19 века. Основным предметом её изучения и тогда являлся и сейчас является аксиоматический метод - совершенно классическая и ключевая часть математики.
Касательно метода, в математике сплошь и рядом на математическом уровне строгости строят модели изучаемых "явлений". Так как математическая логика изучает аксиоматики и связанные понятия, то в ней это все формулируется на математическом уровне строгости и появляются те самые формальные теории и формальные выводы. Дальше про них доказываются теоремы и т.д. Весь этот процесс ничем принципиально не отличается от других разделов. И да, формальные выводы - это не инструмент, а в зависимости от ситуации предмет изучения или вспомогательное понятие, ровно также, как скажем, группы в алгебре.

Касательно доминирования связей с информатикой -хотя они может несколько доминируют над связями с математикой, но мат.логи в любом случае довольгно изолирована и от информатики.
Аноним 01/04/15 Срд 01:36:57 #433 №252464 
>>252461
>о вписывается в алгебру, так что норм её считать математикой, расслабтесь, йоу, получайте удовольствие просто от всего этого.
>Вот то ли дело матстат.
Объясни мне без чисел, почему так получается, что a(b+c)=ab+ac
Аноним 01/04/15 Срд 02:31:26 #434 №252468 
>>252464
ну для этого есть теория операд или теория универсальных алгебр
Аноним 01/04/15 Срд 03:11:39 #435 №252469 
>>252463
Лейбниц сделал много хороших вещей. Например, именно он сделал общеупотребительной арифметическую нотацию: символы +, -, = и прочие. Лейбниц вёл довольно бугуртоёмкие срачи с людьми, которые считали арифметические символы вздором и требовали для записи действий использовать только слова.
Лейбниц мечтал сделать логику наукой, подобной арифметике - чтобы над логическими высказываниями можно было бы совершать действия, похожие на те, что совершаются в арифметике. Именно Лейбниц сформулировал идею формального вывода.
Лейбниц активно исследовал двоичную систему счисления и близко подошёл к идее булевых алгебр, опередив Буля на сто с лишним лет. Эта идея так сильно его занимала, что он даже создал теологическую концепцию "монад" и уверовал в свою монадологию.
Именно Лейбниц придумал термин "модель".

>на математическом уровне строгости
Этот термин подсказывает, что существует более жёсткий уровень строгости, чем математический. Естественно считать, что науки, которые работают на этом более жёстком уровне, не являются частью математики.
Математический уровень строгости можно увидеть в математических доказательствах. Типичное математическое доказательство не является ещё формальным доказательством. Математики чрезвычайно редко пользуются формальным выводом. Даже известный французский генерал, трактат которого здесь считают эталоном заформализованности, на самом деле почти не пользовался формальным выводом.
>доказываются теоремы
Доказываются, что важно, формальным выводом, а не обычными математическими рассуждениями.
>довольно изолирована и от информатики
Информатика - это теория формальных грамматик, теория алгоритмов, теория информации, теория данных, теория знаний (в том числе формальные онтологии) и всё прочее. Математическая логика много заимствует из этих теорий, тогда как её заимствования из математики весьма скромны.
Аноним 01/04/15 Срд 09:37:58 #436 №252495 
>>252434

> Но именно так ты и заявляешь ну или не ты, а другой хуй, который тут всем ебёт мозги основаниями: Логика и метаматематика это все части математики, т.к. без них математики не существует.

Просто ты плохой математик (зубрилко) и не видишь неочевидные связи между понятиями (и то что ты пренебрежительно относишься к основаниям это подтверждает). Врочем и зубрилка ты херовый, так как об этом в литературе по матлогике открытым текстом сказано.

>>252446

Да плевать как ты считаешь (тем более ты показал, что не петришь в математике, так что даже авторитетом не можешь давить), здесь тебе не выборы в городское собрание. Давай доказательства или уёбывай.

Аноним 01/04/15 Срд 09:54:42 #437 №252499 
>>252456

> В математике не используются методы, которые применяются математической логикой. Математики крайне редко строят формальные модели, тогда как главный инструмент математической логики - это именно формальный вывод.

Тебе уже писали, это исключительно технический и психологический момент связанный с колечественной сложностью восприятия формализма. Ну и с тем, что у разных направлений философии основания могут несколько отличаться, поэтому удобнее строить доказательства независимо от них на более высоком уровне.

PS Как мне нравятся эти МАТЕМАТИЧЕСКИЕ поняти я вроде "скорее", "слабо" и т.д.

PPS Ты просто не усовил свои уроки. Мат. логика имеет двойное назначение - как основа CS, так и раздел обоснования математики, об этом обычно во введении к книжкам по матлогике пишут.
Аноним 01/04/15 Срд 10:13:35 #438 №252502 
>> Логика и метаматематика это все части математики, т.к. без них математики не существует.

Что не так в этой фразе???
Вот вам математическое ее доказательство.
Убираем логику. Теперь попробуйте без нее хотя бы что нибудь сформулировать. Ч.т.д.

Берём метаматематику в той её части где рассматриваются исходные понятия. В результае получаем математику которая понятия не имеет на основе чего она строится и почему. Как показала история при этом получаются противоречия.
Аноним 01/04/15 Срд 10:25:42 #439 №252506 
>>252469

>на математическом уровне строгости
>>Этот термин подсказывает, что существует более жёсткий уровень строгости, чем математический.

Нет, математический уровень строгости считается максимальным, просто он может варироваться в зависимости от контекста.

Аноним 01/04/15 Срд 10:28:05 #440 №252507 
>>252495
>>252499
>>252502
Ооо, горе-фейлософ любитель оснований вернулся. Ты скоро станешь тут нарицательным персонажем, наподобие переводчика - что-то вроде местного поехавшего.
Аноним 01/04/15 Срд 10:28:19 #441 №252508 
Я вчера как следует зарядился и мозг задался таки вопросом:
Допустим у нас есть точка, которая находится в пространстве у которого количество измерений стремится к бесконечности.
Можно ли сделать проекцию этой точки на трехмерное пространство и что получится в итоге? Бесконечное множество трехмерных пространств с этой точкой?
Аноним 01/04/15 Срд 10:29:39 #442 №252509 
>>252507

Это всё что может выдать недоматематик-неосилятор? Неудивлён.
Аноним 01/04/15 Срд 10:29:40 #443 №252510 
>>252508
p.s. в математике шарю на уровне таблицы умножения, если что.
Аноним 01/04/15 Срд 10:30:30 #444 №252511 
>>252510

Уже неплохо. Это намного выше уровня вот этого персонажа >>252507
Аноним 01/04/15 Срд 10:30:52 #445 №252512 
>>252509
Ну я просто помню, что проекция четырехмерного объекта в трехмерное выглядит как анимация. Вот и стало интересно
Аноним 01/04/15 Срд 10:32:20 #446 №252513 
>>252512

Хабром повеяло. Но как связано четырехмерное пространство с основаниями математики???
Аноним 01/04/15 Срд 10:38:30 #447 №252515 
14278739107470.gif
>>252513
Ну я о вот этой штуке.
Это ж типа четырехмерный тессеракт спроецированный в трехмерное пространство А потом спроецированный на двухмерную плоскость. Я поэтому и подумал, что если есть точка или фигура в пространстве, в котором множество измерений - бесконечность, то его можно спроецировать на трехмерное пространство, но т.к. множество измерений бесконечное, то и получится должно множество трехмерных пространств, которое бесконечно.
Или стремиться к бесконечности. Я плохо шарю в этих терминах
Аноним 01/04/15 Срд 10:42:28 #448 №252516 
>>252506
Что есть "математический уровень строгости"?
Аноним 01/04/15 Срд 10:45:15 #449 №252517 
>>252516

Формальное доказательство в подходящей ФС. Иначе это неформальное понятие, можно только на пальцах объяснить иначе никак.

Аноним 01/04/15 Срд 10:56:48 #450 №252518 
>>252502
>Убираем логику. Теперь попробуйте без нее хотя бы что нибудь сформулировать. Ч.т.д.
Зачем убирать? Берем нотацию, 3,5 правила и ограничиваемся ими. Тратим на это 5 минут. Этого хватает на всю математическую карьеру, это уровень доказательств любой сложности.
Аноним 01/04/15 Срд 10:58:09 #451 №252519 
>>252518

Да хоть одно правило, это всё равно логика как её не назови.
Аноним 01/04/15 Срд 10:59:14 #452 №252520 
>>252450
Часть теорката. Теоркат использует логику ровно настолько же, как и любой другой раздел математики - на минимум.
Аноним 01/04/15 Срд 11:00:19 #453 №252521 
>>252518

> Этого хватает на всю математическую карьеру, это уровень доказательств любой сложности.

Это мне напоминает того мужика который всю жизнь говорил прозой и не знал об этом. Позиция страуса прячущего голову в песок.
Аноним 01/04/15 Срд 11:05:10 #454 №252522 
>>252519
Только я не знаю ни теорем логики, ни моделей, ни разных логик, ни сугубо логических терминов, ничего от этой науки. Я знаю только И, ИЛИ, импликацию и отрицание.

>>252521
Только страус по легенде прячет в голову песок от опасности. А нормальные люди прячутся от ненужной инфы, которая никогда не пригодится им в работе.
Аноним 01/04/15 Срд 11:12:56 #455 №252524 
>>252518

Во-первых, это не ответ. Как я уже говорил, даже минимальное использование это всё равно использование.

Во-вторых, как ты без логики и серьёзного разбора оснований разрулил бы, например, аксиому произвольного выбора?

>>252522

> Только страус по легенде прячет в голову песок от опасности.

Ключевое там не опасность, а нежелание видеть. Впрочем есть и опасность - противоречия математики.

> А нормальные люди прячутся от ненужной инфы, которая никогда не пригодится им в работе.

Это дети грудного возраста, которые думают если закрыли глаза то их никто не видит. Ну и насчёт того, что люди просто пользуются, это опять же тот герой Мольера.
Аноним 01/04/15 Срд 11:19:19 #456 №252525 
Оу, ответьте мне что-нибудь >>252515
Аноним 01/04/15 Срд 11:42:41 #457 №252531 
>>252524
>Во-вторых, как ты без логики и серьёзного разбора оснований разрулил бы, например, аксиому произвольного выбора?
>произвольного
А есть аксиома непроизвольного выбора? Не знал, не знал.
А так мне плевать вообще, я и так дохуя утверждений на веру принял, мне придется потратить лет 10 своей жизни, чтобы самостоятельно все обосновать и свести к минимальному набору аксиом, а вместо этого я могу просто верить сообществу математиков и потратить эти 10 лет более продуктивнее.

>Впрочем есть и опасность - противоречия математики.
Опасность была и будет всегда, однако это не мешало всем математикам мира работать.

>герой Мольера
Я живу в эпохе постмодерна, где поданная в виде худпроизведения мысль уже не воспринимается как истина в послендей инстанции.
Аноним 01/04/15 Срд 11:43:06 #458 №252532 
>>252524
Фейлософу нужно объяснять его же петушиными методами аналогиями, иначе он не поймет.

Возьмем, к примеру, две такие науки - химию и физику. Ясно, что на фундаментальном уровне все химические взаимодействия можно свести к физическим законам поведения элементарных частиц. Но если внезапно все физики куда-то пропадут, все учебники по физике сгорят, и само слово физика забудут, химические законы те, которые находятся уровнем иерархии выше физических всё равно так же будут работать - обладать предсказательной силой.

Примерно в такой же зависимости друг от друга находятся логика и математика.
Аноним 01/04/15 Срд 11:56:02 #459 №252536 
>>252469
>>на математическом уровне строгости
>Этот термин подсказывает, что существует более жёсткий уровень строгости, чем математический.
В основном в теории и в работах по пруверов (а это раздел информатики к которому кстати недавно потянулись алг. топологи и т.п.). Открой более менее произвольную статью по логике и обнаружишь там нормальные матемтические доказательства с некоторыми недоговорками, опусканием очевидной специалистам рутины и т.п.
>Математическая логика много заимствует из этих теорий, тогда как её заимствования из математики весьма скромны.
Довольно ограниченно - куда более характеерно использование логики в информатике, чем наоборот. Я бы сказал, что использование в логике таких вещей, как общая топология и теория категорий куда более существенны, чем наработок из информатики и лингвистики; исследований по алгоритмической сложности логик, например, конечно куча, но не припомню ни одного случая, когда это использовалось для чего либо кроме подсчета алг. сложности других логик.
Аноним 01/04/15 Срд 11:56:25 #460 №252537 
>>252469
>>на математическом уровне строгости
>Этот термин подсказывает, что существует более жёсткий уровень строгости, чем математический.
В основном в теории и в работах по пруверов (а это раздел информатики к которому кстати недавно потянулись алг. топологи и т.п.). Открой более менее произвольную статью по логике и обнаружишь там нормальные матемтические доказательства с некоторыми недоговорками, опусканием очевидной специалистам рутины и т.п.
>Математическая логика много заимствует из этих теорий, тогда как её заимствования из математики весьма скромны.
Довольно ограниченно - куда более характеерно использование логики в информатике, чем наоборот. Я бы сказал, что использование в логике таких вещей, как общая топология и теория категорий куда более существенны, чем наработок из информатики и лингвистики; исследований по алгоритмической сложности логик, например, конечно куча, но не припомню ни одного случая, когда это использовалось для чего либо кроме подсчета алг. сложности других логик.
Аноним 01/04/15 Срд 11:59:22 #461 №252540 
>>252536
>Я бы сказал, что использование в логике таких вещей, как общая топология и теория категорий куда более существенны, чем наработок из информатики и лингвистики;
Тарский встал и вышел.
Аноним 01/04/15 Срд 12:04:12 #462 №252543 
>>252540
Скажем Тарскому "гудбай".
Аноним 01/04/15 Срд 12:04:47 #463 №252544 
>>252540
Великий логик, не в пример тому же разрекламированному Тьюрингу.
Аноним 01/04/15 Срд 12:05:07 #464 №252545 
>>252543
Да здравствуют сантехники!
Аноним 01/04/15 Срд 12:05:41 #465 №252546 
>>252545
Кто не Тарский, тот сантехник.
Аноним 01/04/15 Срд 12:06:30 #466 №252547 
>>252532

Ну просто сказочный долбоёб, сочиняет аналогии которые бьют его же самого. Без физических представлений химия это примитив вроде алхимии средних веков с магами и шлюхами.

Аноним 01/04/15 Срд 12:06:59 #467 №252548 
>>252544
Ты дурак что-ли????? тарский ваще ссаный философ бля гуманитарий он язык изучал!! он ваще нихуя в логику не привнес, так бы и сосали хуй пока не пришли ребята типа гротендика и не привнесли революцию в логику!!!
Аноним 01/04/15 Срд 12:09:14 #468 №252549 
>>252547
>Без физических представлений химия это примитив вроде алхимии средних веков с магами и шлюхами.
Так оно и есть, однако это не мешает развиваться науке, даже несмотря на то, что даже самую простую химическую реакцию пока что смоделировать нельзя.
Аноним 01/04/15 Срд 12:10:22 #469 №252550 
>>252502
Математика прекрасно существовала до математической логики, её результаты в целом были достоверны еще по крайней мере со времен Евклида. Некоторые проблемы с нехваткой строгости конечно бывали, но катастроф не происходило, даже Кантор с явно парадоксальным учением о множествах, на сколько мне известно, не доказал ничего противоречащего современным представлениям.
Аноним 01/04/15 Срд 12:17:08 #470 №252552 
>>252531

> А есть аксиома непроизвольного выбора? Не знал, не знал.

Без разбора оснований ты бы даже задуматься о ее существовании не смог. Считал бы очевидным утверждением.

> А так мне плевать вообще, я и так дохуя утверждений на веру принял, мне придется потратить лет 10 своей жизни, чтобы самостоятельно все обосновать и свести к минимальному набору аксиом, а вместо этого я могу просто верить сообществу математиков и потратить эти 10 лет более продуктивнее.

Я говорю о математике как о науке, а не о том что тебе хочется или не хочется учить. Мне например геометрия не нравится, я же не ору, что она не нужна и не является разделом математики.

С основаниями ситуация немного отличается от спец разделов, поскольку они находятся внизу, в самом начале. А значит все последующие разделы зависят от этих оснваний. И игнорировать их это значит быть непоследовательным и просто обманывать себя. Такой подход пару веков назад канал, но не сейчас.

> Опасность была и будет всегда, однако это не мешало всем математикам мира работать.

Доработались до кризиса, пришлось учить основания.

> Я живу в эпохе постмодерна, где поданная в виде худпроизведения мысль уже не воспринимается как истина в послендей инстанции.

Это не мысль, а комический персонаж который занимаелся ровно тем же чем и ты.

Аноним 01/04/15 Срд 12:17:56 #471 №252553 
>>252548
Фактически Тарский придумал теорию моделей и за ним много несколько менее значительных достижений. Когда ты говоришь про Гротендика и логику ты наверное имеешь в виду топосы; кстати в логику их привнес не сам Гротендик, а Лавер. Топосы это, конечно занятно, в свое время было довольно модно, но до революционности это вовсе не дотягивает.
Аноним 01/04/15 Срд 12:21:01 #472 №252554 
>>252550

Ну попробуй опубликуй статью с доказательствами при помощи одних рисунков. То о чем ты говоришь, это история математики. К обсуждаемому вопросу вообще никакого отношения не имеет.
Аноним 01/04/15 Срд 12:21:20 #473 №252555 
>>252552
>Мне например геометрия не нравится, я же не ору, что она не нужна и не является разделом математики.
Только если я хочу заниматься другой областью, но мне понадобиться геометрия, я буду обязан ее изучать. Т.к. она прямо связана чуть ли не со всеми областями математики. А логика мне вообще не нужна, кроме базовых 3,5 утверждений, принятых на веру.

>С основаниями ситуация немного отличается от спец разделов, поскольку они находятся внизу, в самом начале.
Нет, это средневековый подход. Они находятся в одной плоскости со всеми остальными науками.

>Доработались до кризиса, пришлось учить основания.
Кризис продолжается до сих пор, я напомню, что на него просто забили. Основания не оправдали себя.

>Это не мысль, а комический персонаж который занимаелся ровно тем же чем и ты.
Почему он комический? Потому что не соответствует средневековым нормам общества?
Аноним 01/04/15 Срд 12:22:07 #474 №252556 
>>252555
>Кризис продолжается
Ну и в чём же это проявляется?
Аноним 01/04/15 Срд 12:30:04 #475 №252562 
>>252554
История имеет значения и показывает, что никакое специальное изучение логики для занятия математикой не нужно.

Ну на рисунках наверное не опубликую. Но и изучать для логику для написания современных мат. статей тоже не обязательно. Конечно можно сказать, что современной культуре математической строгости математика до некоторой степени обязана логике. Но эта культура не определяюща для математики - математика существовала бы и на несколько меньшем уровне строгости.
Аноним 01/04/15 Срд 12:32:20 #476 №252563 
>>252555

> Только если я хочу заниматься другой областью, но мне понадобиться геометрия, я буду обязан ее изучать.

Нет, по твоему же подходу если я использую только интуитивно понятные геометрические образы которые риусю на бумажке, мне ничего учить и доказывать не надо.

> Нет, это средневековый подход. Они находятся в одной плоскости со всеми остальными науками.

Это не подход, а факт. Асли А оснвано на B, но не ноборот, то B является основанием для А.

> Кризис продолжается до сих пор, я напомню, что на него просто забили. Основания не оправдали себя.

Как оснвоание чего-то может оправдать себя или нет? Основание всегда есть (другое дело его можно знать или не знать). Ты походу опять перепутал с программами по основаниям математики.

Аноним 01/04/15 Срд 12:38:46 #477 №252570 
>>252553
Теория моделей это изучение языка в чистейшем виде, лингвистика формальных языков.

>>252556
В том, что он не преодолен, все программы его преодоления зафейлены.

>>252563
>Нет, по твоему же подходу если я использую только интуитивно понятные геометрические образы которые риусю на бумажке, мне ничего учить и доказывать не надо.
С чего вдруг? А если алгебраические методы более продуктивны? В то же время изучение оснований мне ничего не даст.

>Это не подход, а факт. Асли А оснвано на B, но не ноборот, то B является основанием для А.
Что первично в таком случае: геометрия или алгебра?

> Ты походу опять перепутал с программами по основаниям математики.
Не суть важно. Они не оправдали себя. Основания это бессмысленные конструкции в которые на данный момент можно только верить, кто-то в Будду, кто-то в Аллаха.
Аноним 01/04/15 Срд 12:47:37 #478 №252573 
>>252570
>Теория моделей это изучение языка в чистейшем виде, лингвистика формальных языков.
Очень странный тезис. С тем же успехом можно было бы назвать теорию групп лингвистикой групповых тождеств.
Аноним 01/04/15 Срд 12:54:28 #479 №252574 
>>252573
А чем по-твоему занимаются лингвисты? Классифицируют слова по группам и ищут подлежащее со сказуемым?
Аноним 01/04/15 Срд 13:02:48 #480 №252577 
>>252570
>что он не преодолен
Ну а я скажу, что преодолён. И как выяснить, кто прав?
Аноним 01/04/15 Срд 13:11:38 #481 №252581 
>>252570
>все программы его преодоления зафейлены
Это не так, программа Гильберта завершена успешно. Непротиворечивость арифметики доказана неарифметическими средствами. Конкретнее, непротиворечивость аксиом Пеано доказал в 1936 году Генцен с ипользованием трансфинитной индукции.
https://en.wikipedia.org/wiki/Gentzen%27s_consistency_proof
Аноним 01/04/15 Срд 13:16:44 #482 №252584 
>>252581
>программа Гильберта завершена
Бля, я не могу уже с этого треда. Хотя она действительно завершена: жирный крест на ней поставил Гедель.

>>252577
Почитать Куна, например.
Аноним 01/04/15 Срд 13:20:58 #483 №252585 
>>252574
Когда центральной темой оказывается не язык, а семантика, при этом сам язык оказывается на перефирие - это уже не лингвистика. Если что-то уж называть лингвистикой формальных языков, то теорию доказательств. Впрочем, не вижу в теории доказательств ничего плохого - тоже замечательный раздел логики.
Аноним 01/04/15 Срд 13:22:33 #484 №252586 
14278837535760.jpg
>>252552
>поскольку они находятся внизу, в самом начале. А значит все последующие разделы зависят от этих оснваний.

Kek, тогда давай начнем изучение математики с семантики, этимологии и правил словообразования. Мы же всё пользуемся языком для оформления своих мыслей, и не хотим быть в глупой ситуации, - схожей с мольеровской, - оказаться незнающими законов словообразования собственного языка. А вдруг этимология противоречива? хотя она очевидно противоречива Тогда придется отбросить математику и сидеть ждать пока кто-нибудь придумает новый усовершенствованный язык.
Аноним 01/04/15 Срд 13:23:16 #485 №252587 
>>252585
Ну все равно это раздел семиотики.
Аноним 01/04/15 Срд 13:24:23 #486 №252588 
>>252584
>Кун
>давно усторевшее говно
>Почитать Куна
Ох лол.
Аноним 01/04/15 Срд 13:25:17 #487 №252589 
>>252588
>устаревшее
фикс лол
Аноним 01/04/15 Срд 13:25:44 #488 №252590 
>>252586
>и не хотим быть в глупой ситуации, - схожей с мольеровской, - оказаться незнающими законов словообразования собственного языка.
Да уж, неприятная оказия! Как говорил великий Мольер "От книжной мудрости глупец тупее вдвое."
Аноним 01/04/15 Срд 13:25:58 #489 №252591 
>>252584
>жирный крест на ней поставил Гедель
Чушь. Гёдель лишь получил один из важных результатов. Окончательно программу завершил Генцен.
Аноним 01/04/15 Срд 13:26:47 #490 №252592 
>>252588
>имплаинг обоснования глупости вопроса "кто прав" требует каких-то невероятно свежих знаний о научных теориях
Аноним 01/04/15 Срд 13:29:05 #491 №252593 
>>252592
>имплаинг Кун дает какое- то знание о реальных научных теориях
Кек.
Аноним 01/04/15 Срд 13:30:09 #492 №252594 
>>252591
Т.е. ты серьезно отстаиваешь тезис программа гильберта завершена? Без шуток? Т.е. доказано, что все математические теории финитны и кончаются определенным набором непротиворечивых аксиомам? А то, что непротиворечивость этих аксиом нельзя доказать, это важный результат этого великого доказательства? Ну нихуя себе!
Аноним 01/04/15 Срд 13:30:37 #493 №252595 
>>252593
Сантехник, не узнаю вас в гриме.
Аноним 01/04/15 Срд 13:33:16 #494 №252597 
>>252592
Ну тащемто как- то всерьез советовать Куна- это какгбе очевидный зашквар и характернейший признак безграмотной гуманитарной маньки.
Аноним 01/04/15 Срд 13:43:14 #495 №252599 
>>252594
Да, программа Гильберта завершена. Все математические теории можно известным образом свести к формальным системам, формальные системы, в свою очередь, метаматематически можно понимать как утверждения о натуральных числах. А непротиворечивость аксиоматики Пеано доказана.
Аноним 01/04/15 Срд 13:45:39 #496 №252600 
>>252587
Не собираюсь спорить о чём-то, о чём имею довольно слабое представление (семиотика) - не исключаю, что теория моделей развивает какие-то идеи появившиеся в её контексте. Но вообще теория моделей по своему предмету более всего напоминает алгебру, понимаемую, как науку об алгебраических структурах, и с существенными натяжками может считаться одной из её частей
Аноним 01/04/15 Срд 13:47:09 #497 №252601 
>>252595
Много раз уже видел этот форс про сантехника и у меня сложилось впечатление, что его употребление выдает в форсере неосведомленного фантазера, испытывающего некие неудобства от столкновения с реальностью и называния вещей своими именами. Вот ты только подтвердил это впечатление. То, что теории Куна устарели и много раз обоснованно критиковались- давно уже общее место в современной истории науки, не понимаю почему это вызывает у тебя столько боли.
Аноним 01/04/15 Срд 13:51:48 #498 №252602 
>>252599
Ловите поехавшего.
Как ты, дурачёк, ZFC будешь сводить к "утверждениям о натуральных числах", к примеру?
Аноним 01/04/15 Срд 13:55:24 #499 №252603 
>>252602
Ты читал трактат Бурбаки? В нём развивается система аксиом, эквивалентная системам аксиом Цермело-Френкеля с аксиомой глобального выбора. В первом томе вполне естественным образом, в ходе определения понятия "формальная математическая теория", осуществляется это сведение. Метаматематическое, а не математическое, ещё раз обращу внимание.
Аноним 01/04/15 Срд 13:56:12 #500 №252604 
>>252602
Ну здесь он более менее прав. Утверждение о непротиворечивости ZFC, как учит нас Матиясевич, эквивалентно отсутствию корней у некоторого явно выписываемого диофантового уравнения; аналогично утверждения о доказуемости каких-то теорем эквивалентно переписываются, как утверждения о существование корней у диофантовых уравнений.
Аноним 01/04/15 Срд 13:58:27 #501 №252605 
>>252601
>неосведомленного фантазера, испытывающего некие неудобства от столкновения с реальностью и называния вещей своими именами.
This. Если чего- то нет, но очень хочется, чтобы существовало- феласофствуй и называй всех кто не согласен сантехниками, ага. Форс про сантехника придумал типичный Алешенька из винрарной пасты.
Я тут размышлял о характерном споре аметистов и верунов, где с аметистовой стороны приводятся простые, банальные даже, доступные каждому школьнику иллюстрации идей: чайник Расселла, невидимый розовый единорог, ЛММ. А веруны, конечно не самые простые, а которые пообразованнее, говорят с ленцой: ну это всё детский сад, богословие имеет историю глубокой философской мысли, об этом библиотеки написаны. Поэтому возразить на ваши чайники по сути нечего, но это как бы нечестный ход, удар кувалдой по шахматной доске. Лучше почитайте Платона, Аристотеля тож, отцов церкви, и приходите играть с нами в наши игры. Аметисты обижаются, не приходят.

А вот мне пришла в голову любопытная аналогия.
Представим себе спальный район обычного рабочего города. Живут там два друга детства, пошедшие разными путями. Один - культурнейший, интеллигентный христианин, мыслитель, поэт и тонкая душа, похожий на А.Ф. Лосева в молодости, тратящий все деньги на редкие издания классической литературы. Другой - быдловатый хуйлан в абибасе, Бертран Сиплый, прочитавший за всю жизнь четыре книжки: букварь, зелёную с пятном кофе, "Слепой против Бешеного" Очобы и "Бог как Иллюзия" Докинза. Общаются они теперь, понятное дело, нечасто, но иногда Лосев, возвращаясь из букинистического магазина, великодушно пожимает руку сидящему с друганами у падика на кортах Бертрану. И тот, в свою очередь, относится к рассеянному товарищу со смесью превосходства и восхищения: дохляк дохляком, но какие телеги загонять умеет!

Годы идут своим чередом, и молодой Лосев влюбляется в девушку Наташу. Наташа наполовину еврейка, наполовину мордвинка, имеет большую грудь, уступчива, крашена в блондинку и громко смеётся. Её наивная первородная витальность очаровывает Лосева, да так, что он всерьёз подумывает о женитьбе. Только вот Бертран Сиплый, заметив это, как-то подходит переговорить и доверительно сообщает:
– Друг, ты чо ёбнулся? Ты посмотри на неё, это ж наша районная сиповка Натка Сквозная, её вчера вон те хачи пёрли в три смычка! Глянь, да у неё в патлах малафья засохла! Очевидно же всё!
Лосев корректным, но решительным образом выражает своё нежелание слушать подобную мерзость и удаляется. Как назло, тем же вечером, сливаясь с Наташей в поцелуе, он ощущает привкус спермы на языке (откуда этот вкус ему известен – не спрашивайте). Конечно, сам он не смел бы просить Наташу о "французской любви", поэтому возникают некоторые мысли. Лосева снедает странная вяжущая тоска, и в поисках избавления он, прополоскав рот, обращается к домашней библиотеке. На середине очередного монумента по христианской семиотике его посещает оригинальная идея, и к утру готов манускрипт статьи: "Прекрасная Дама как Вечное Сомнение: энтелехия греха, очищение и Любовь в христианской алхимии". Удовлетворённый ответом на грани человеческого и божеского разумения, Лосев засыпает в кипе бумаг.

На другой день Наташа опаздывает в музей истории религии. Прождав два часа, Лосев едет домой и видит, как её на скамеечке окружили азербайджанцы с похотливыми и тупыми лицами. Наташа раскраснелась и смеётся до слёз, Наташе хорошо. Изумлённый Лосев, крикнув "отойдите от моей девушки немедленно!", врывается в стаю южан и просыпается на больничной койке. Рядом - друг с парой свежих швов на лице и сбитыми костяшками. Прихлебнув из эмалированного чайника с рисунком Марса, Бертран кашляет и сипло говорит:
– Братан, бля буду, не путайся ты с ёбаной шмарой, дороже выйдет. Кабы я не шёл с пацанами из пивного, тебя бы эти волки замесили нах. Она ж тебе не пара, она по всем хуя-
– Не трать силы, меня не интересует эта приземлённая грязь, которая тает от одного лишь света моей Дамы, воплощённого Знания! Только из уважения к нашей дружбе я не велю тебе выйти немедленно!
– Бля, Лёха, ну ты и лось. Ну хорошо... бля, как это сказать... объяснить попроще...
На низком лбу Бертрана от непривычного труда показывается пот. Кусает губы, тяжело дышит. Неопределённое число вздохов спустя он, наконец, выдаёт:
– Слышь, короче... Это как в зелёной книжке сказано. Эти твои Софии, Прекрасные Дамы, традиционализм, алхимия эта ёбаная, каббала, платонизм, патристика, апофатика-хуятика, это всё замкнутые системы. Натки Сквозной может вообще не быть, ты можешь дрочить всю жизнь на фантазии о богочеловечестве или точке Омега, сидя в комнате. А сейчас что есть она, что её нет, но любишь-то ты не Натку, а ту сложную херотень из слов, которую придумал, читая книжки. Я, Лёх, понимаю, что это наверное красиво, но зачем же тебе коза гулящая с её еврейскими суевериями, с глупостью, с заебонами деревенскими, зачем из-за неё пизды получать, скажи, а? Зачем же ты запомоился об эту блядь, которой даже я под портвешок гузно шатал? Энтелехия у тебя в башке и в книжках, а во рту-то малафья, тут никакая теодицея не поможет! – И чайник со стуком опускается на прикроватный столик.
Лосев неспокоен, в смятении. Он не ждал такого и в то же время догадывался сам. Что, если друг Бертран, при всей низости своих доводов, прав, прав фактически, истинно и безусловно – то есть нет ни одной реальной причины над сиповкой Наткой додумывать Вечную Женственность Настасию? Что, если факты есть факты, а символы есть символы, и платонизм тут уже не при делах? Что, если его работы, его будущие монографии, его именные кирпичики в здании христианской мысли – не более чем игровое бисероплетение дырками от бубликов, химера, сексуальная перверсия маленькой кучки гуманитариев-эрудитов, иначе говоря фуфло? Но вдруг какая-то сила охватывает его властно, наливает члены дрожью, и, сам того не ожидая, Лосев отвечает с перекошенной ухмылкой:
– Аметисторебёнок бомбанул, смотрите братия, такая-то РЕФОРМАЦИЯ. Читай Платона! Бульвинкль, куклачую капчу! Борода, Собянин, смерть! АЗАЗА!
Бертран смотрит в выписку, вчитывается в корявый врачебный почерк: «Закрытая ЧМТ, кровоизлияние в левой теменной доле, возможны осложнения». Ему понятно только, что друга у него больше нет. Вздыхает ещё раз, грустно надевает кепку, выходит. В дверях палаты оборачивается:
– А да и хуй с тобой. По-любому, как ты энтелехию в рот брал, я тебе и руки не подам. Дурак ты, Лёха, хотя и умный. Ну живи как знаешь.

Больничная дверь со скрипом, аккуратно, прикрывается. Чайник остаётся стоять. На фоне Марса, если приглядеться, видна крошечная Наташа с энигматической улыбкой Девы Марии, в белой маечке ΙΧΘΥΣ.

Занавес.
Аноним 01/04/15 Срд 14:07:49 #502 №252607 
Вам делать нехуй что ли, ебать, или че? Какого хуя вы в мат-тред претесь ебать? Я вас, пидорасов, гнобил и буду гнобить нахуй, пожизняк ебать, с 1 курса ебать. Убивал и буду убивать нахуй.
Аноним 01/04/15 Срд 14:19:42 #503 №252609 
>>252603
>>252604
Ну-ка, сведите мне ZFC-шную аксиому power-set'а к аксиомам из PA.
В ZFC можно сформулировать и доказать утверждения, которые нельзя доказать посредством PA (Теорема Гудстейна, например). Блять, это же хуй знает когда уже доказано, в помощью моделей.
Аноним 01/04/15 Срд 14:25:31 #504 №252614 
>>252609
Так я и не говорю, что он во всем прав - в целом он несет чушь, не понимая существа программы Гильберта. Но вот это утверждение
>Все математические теории можно известным образом свести к формальным системам, формальные системы, в свою очередь, метаматематически можно понимать как утверждения о натуральных числах.
совершенно верно.
>В ZFC можно сформулировать и доказать утверждения, которые нельзя доказать посредством PA (Теорема Гудстейна, например).
Утверждения о натуральных числах =/= утверждения доказуемые в PA.
Аноним 01/04/15 Срд 14:27:19 #505 №252615 
>>252614
О существе программы Гильберта я бы подискутировал. Каково твоё понимание этой программы?
Аноним 01/04/15 Срд 14:27:37 #506 №252616 
>>252614
>совершенно верно.
здесь я наверное погорячился - сформулировано оно несколько коряво и способствует некорректному пониманию, но в любом случае случае в целом оно верно.
Аноним 01/04/15 Срд 14:32:27 #507 №252618 
>>252607
Лол, ребята наверно сидели лампово, обсуждали теорему Гёделя с её далеко идущими и значительными для всего философского мира следствиями. Тут втиснулась какая-то япошка понаехавшая, начала пидорашками обзывать. Я бы тоже не стерпел.
Аноним 01/04/15 Срд 14:33:31 #508 №252619 
>>252615
Гильберт хотел построить формализовать математику, далее доказать полноту и непротиворечивость этих систем финитными методами. Стоит отметить что если бы это оказалось верно, то автоматически все абстрактные методы оказались бы консервативны над финитной математикой. Собственно полнота не вышла совсем, а для непротиворечивости нельзя ограничится финитными методами. То есть логика оказалась устроена совсем не так, как думал Гильберт.
Аноним 01/04/15 Срд 14:35:46 #509 №252622 
>>252619
>Гильберт хотел построить формализовать математику, далее доказать полноту и непротиворечивость этих систем финитными методами.
-->
>Гильберт хотел формализовать математику, построив подходящие аксиоматики и системы вывода, далее доказать полноту и непротиворечивость этих систем финитными методами.
Аноним 01/04/15 Срд 14:44:07 #510 №252627 
У меня от вашей математики жизнь по пизде пошла. Медленно, но тенденция однозначная.
Аноним 01/04/15 Срд 14:48:47 #511 №252632 
14278889277320.jpg
>>252616
Не погорячился, а хуйню сморозил. Модель PA вложена в модель ZFC (см. пик).
Аноним 01/04/15 Срд 14:53:19 #512 №252634 
>>252632
И что? Модели моделями, но вообще ZFC формальная теория, описываемая схемами аксиом и правилами вывода - все это вполне финитные объекты, которые при желание можно кодировать натуральными числами.
sageАноним 01/04/15 Срд 14:54:24 #513 №252635 
Мартышки продолжают стремиться к иерархии.
Аноним 01/04/15 Срд 14:55:20 #514 №252636 
>>252632
Картинка кстати дурацкая - она не отличает теоремы, которые не являются частью PA т.к. они сформулированы в языке теории множеств и арифметические теоремы, которые PA не доказывает.
Аноним 01/04/15 Срд 15:01:00 #515 №252637 
>>252619
В такой формулировке программу Гильберта следует считать провалившейся, согласен.

Давай обозначим то, что ты описал, символом T.
Я понимаю программу Гильберта так.
1. Выяснение возможности совершить T целиком либо частично.
2а. В случае, если совершить T целиком возможно, совершение T целиком.
2b. В случает, если совершить T целиком невозможно, но совершить T частично всё же возможно, совершение по возможности большей части T.
Аноним 01/04/15 Срд 15:10:02 #516 №252638 
>>252562

> История имеет значения и показывает, что никакое специальное изучение логики для занятия математикой не нужно.

С точки зрения математики история значения не имеет, важен результат. Сейчас всем плевать на Лейбницевские бесконечно малые, например, все рассуждают на няшном языке "эпсилон-дельты" и не используют какую-то неведомую херню.

> Ну на рисунках наверное не опубликую. Но и изучать для логику для написания современных мат. статей тоже не обязательно. Конечно можно сказать, что современной культуре математической строгости математика до некоторой степени обязана логике.

Не некоторой степени обязана, а полностью. Любое содержательное доказательство должно допускать перевод на формальный язык, это неявно предполагается. Это как машинный код и языки высокго уровня вроде C# или Haskell. Можно всю жизнь писать только на них и не иметь представления об архитектуре компа на низком уровне. Тем не менее внутренне все конструкции языков программирования высокого уровня в конечном счете сводятся к машинным кодам.

> Но эта культура не определяюща для математики - математика существовала бы и на несколько меньшем уровне строгости.

Тогда бы её статус вкорне поменялся и был бы опущен до уровня естественных наук требующих опыта для обоснования результатов.

>>252563

> Что первично в таком случае: геометрия или алгебра?

Как законченые теории они независимы. Тогда как без логического аппарата и основных понятий не обходится ни одна теория.

> Не суть важно. Они не оправдали себя. Основания это бессмысленные конструкции в которые на данный момент можно только верить, кто-то в Будду, кто-то в Аллаха.

Ты в любом случае должен во что-то верить, поэтому и надо определиться с полным списком исходных понятий чтобы двигаться дальше (и желательно их уменьшить их число). Ты же предлагаешь вообще забить на все и заниматься хер пойми чем.

>>252586

По большому счету так и поступают (это и есть явное указание исходных понятий которые не определяются). Короче, иди читай учебники по матлогике, хуйло безграмотное. Тебя ещё в прошлом треде послали.


Аноним 01/04/15 Срд 15:11:22 #517 №252640 
>>252638
>все рассуждают на няшном языке "эпсилон-дельты"
>все рассуждают
>на няшном
С праздником тебя.
Аноним 01/04/15 Срд 15:11:37 #518 №252641 
>>252619

Отрицательный результат, тоже результат. А главное, он ни в коей мере не отменяет необходимости точного определения основ.
Аноним 01/04/15 Срд 15:15:09 #519 №252642 
>>252636
>они сформулированы в языке теории множеств
Это МОДЕЛИ - абстрактные объекты, которые являются своего рода аналогами исходных теорий. Всё сформулировано на одном языке - языке теории моделей, поэтому говорить о таким вложении корректно.

>при желание можно кодировать натуральными числами.
Ну давай уже ближе к делу, "закодируй" мне аксиому power-set'а аксиомами из PA.
Аноним 01/04/15 Срд 15:15:39 #520 №252643 
>>252640

Спасибо.
Аноним 01/04/15 Срд 15:24:27 #521 №252646 
>>252638
>это и есть явное указание исходных понятий которые не определяются
Это есть неоправданное усложнение и перегруженность изложения неработающим материалом. Я понимаю, что для фейлософов - это норма и хлеб насущный, но в математике таких гонят ссаными тряпками.
Аноним 01/04/15 Срд 15:26:16 #522 №252648 
>>252646

С точки зрения малолетнего долбоёба - безусловно.
Аноним 01/04/15 Срд 15:33:11 #523 №252650 
>>252648
Скрылось солнце в облачках.
Съеби, мудило.
Аноним 01/04/15 Срд 15:35:49 #524 №252651 
>>252650

КЕК, хуй тебе в рыло.
Аноним 01/04/15 Срд 15:54:44 #525 №252659 
>>252638
>все рассуждают на няшном языке "эпсилон-дельты"
Чет проиграл с этого математика.
Аноним 01/04/15 Срд 16:02:01 #526 №252661 
>>252597
>гуманитарной
Тарский тоже философ.
Аноним 01/04/15 Срд 16:04:42 #527 №252662 
>>252661

Не пизди. http://en.wikipedia.org/wiki/Tarski%E2%80%93Grothendieck_set_theory
Аноним 01/04/15 Срд 16:06:08 #528 №252663 
>>252662
Тарский и философ и логик. Тем не менее трубы он не чинит, в отличии от тебя.
Аноним 01/04/15 Срд 16:07:33 #529 №252664 
>>252663

Если я прочистил трубу этому обмудку >>252646, это не значит что я её починил.
Аноним 01/04/15 Срд 16:07:40 #530 №252665 
>>252638
>. Можно всю жизнь писать только на них и не иметь представления об архитектуре компа на низком уровне. Тем не менее внутренне все конструкции языков программирования высокого уровня в конечном счете сводятся к машинным кодам.
Аналогия не совсем верна. Тебе предлагают изучить не только ассемблер, но еще и устройство процессора, после очевидно устройство транзистора, p-n переход, квантовую механику... Иначе ты не сможешь дальше писать свой код!
Аноним 01/04/15 Срд 16:09:00 #531 №252666 
>>252665

Она верна ровно в той части, которую я обозначил. Физика тут не важна, так как я имел ввиду только абстрактные машины.
Аноним 01/04/15 Срд 16:12:06 #532 №252668 
>>252638
> Ты же предлагаешь вообще забить на все и заниматься хер пойми чем.
Нет. Я предлагаю поверить в бога, при этом не конкретизируя какого. ZFC так ZFC, хуй с ним, не буду вдаваться в гносеологию, у других вроде работает. Если бы я работал в области теорката или какой-нибудь хоть сколько-нибудь близкой к матлогике, то я бы, наверное, что-нибудь да изучил, а так повторяю, абсолютно нет никакого смысла.
Аноним 01/04/15 Срд 16:14:21 #533 №252669 
>>252665

Перечитал ещё раз что ты написал и не понял а каким образом это опровергает мой тезис? Я же не говорил, что ты не сможешь кодить (как раз наоборот). Но хороший кодер как раз таки должен знать что там происходит внутри, так же и с математикой.
Аноним 01/04/15 Срд 16:18:09 #534 №252670 
>>252642
>Это МОДЕЛИ - абстрактные объекты, которые являются своего рода аналогами исходных теорий. Всё сформулировано на одном языке - языке теории моделей, поэтому говорить о таким вложении корректно.
То что ты говоришь - это неадекватно нагромождения терминов, показывающее твое полное профанство в обсуждаемом вопросе. Кстати, я то подумал, что твоя картинка изображает теории, как множества теорем - все оказалось куда хуже. Объяснять тебе что-то ещё я не намерен - иди почитай учебник по логике.
Аноним 01/04/15 Срд 16:18:14 #535 №252671 
>>252669
>Но хороший кодер как раз таки должен знать что там происходит внутри, так же и с математикой.
С чего вдруг он должен знать? Это "аксиома хорошего кодера" включенная с 2015 в аксиоматику или очередной мещанин во дворянстве?
Аноним 01/04/15 Срд 16:21:14 #536 №252673 
>>252666
Так давай к конкретике, приводи мне ГДЕ ИМЕННО необходимы знания помимо базы логики и базы теорката (можно дать в нескольких главах, как у Бурбаки). В какой области, в какой теореме, в каком месте этой теоремы, только очевидно это не должно быть связано с метаматематикой, теоремы о множествах тут не пройдут.
Аноним 01/04/15 Срд 16:26:13 #537 №252678 
>>252637
Я сам Гильберта почти не читал, но судя по отзывам людей, которые его читали, он всерьез не рассматривал возможности своего провала.
Вся эта история с реформированной программой Гильберта безусловно существует, люди занимались и занимаются связанными с ней вопросами, но это не то, о чем говорил Гильберт.
>>252641
Не спорю, в рамках работы по этой программе было получено много всего полезного.
Аноним 01/04/15 Срд 16:36:56 #538 №252681 
>>252671

Чтобы не быть кодирующей обезъянкой. Это аксиома. А мещанин это из другой оперы. Это где ты обосрался с использованием языка исходных понятий и способов логического вывода.

>>252673

Я уже сто раз здесь писал, что матлогика дает только исходные понятия и средства вывода (большего от нее не требуется). При чем тут знания в каких-то специальных разделах математики, если речь изначально была о её фундаменте?





Аноним 01/04/15 Срд 17:00:04 #539 №252690 
14278968046080.png
14278968046081.png
Аноны, поясните, почему вопрос на пикрелейтед так сформулирован и почему такое решение странное? Я бы задал вопрос так: является ли каждая точка открытого множества E (в любом метрическом пространстве, а не только в R2) предельной точкой E? Ответ: да, потому что каждая точка входит в E вместе с некоторой окрестностью, а значит любая ее окрестность содержит точки E.
Аноним 01/04/15 Срд 17:03:47 #540 №252691 
>>252690
Бывают метрические пространства в которых есть точка и окрестность этой точки, состоящая только из самой этой точки. Поэтому в произвольных метрических не пройдет.
Аноним 01/04/15 Срд 17:08:19 #541 №252694 
>>252691
Это какие, например? С целочисленными координатами?
Аноним 01/04/15 Срд 17:16:19 #542 №252698 
>>252681
Фундамент это 3,5 утверждения, принятых на веру. Ты предлагаешь больше.
Аноним 01/04/15 Срд 17:16:19 #543 №252699 
>>252694
Да, примеров очень много, но например Z^2 с любой из стандартных метрик подходит - в нём вообще все точки изолированы.
В этом контексте мне кажется поучительным понятие разреженных пространств - топологические пространства в которых каждое множество содержит изолированную точку.
Аноним 01/04/15 Срд 17:17:05 #544 №252700 
>>252670
>сам обосрался - спихни на другого
Лол
Аноним 01/04/15 Срд 17:17:19 #545 №252701 
>>252681
>Чтобы не быть кодирующей обезъянкой. Это аксиома.
Ну ясно. Слепая вера обоссаного первокультурника в труъ путь кодера и труъ путь математика.
Аноним 01/04/15 Срд 17:25:28 #546 №252703 
>>252698

define утверждение
Аноним 01/04/15 Срд 17:39:25 #547 №252706 
>>252701

На самом деле всё ещё хуже. Программистам до некоторого предела простительно не знать деталей реализации, собственно для этого и строятся языки высокого уровня (и вообще это простой случай - ассемблер и язык высокого уровня это всё формальные системы одного рода). Тогда как математик не умеющий видеть общую картину и интересующийся только прикладниной это гнилой пидорас, а не математик.
Аноним 01/04/15 Срд 17:40:12 #548 №252707 
>>252706

Потому что математика это не инженерная дисциплина а претендует на всеобщее знание.
Аноним 01/04/15 Срд 18:04:12 #549 №252713 
>>252706
Боль первокультурника. Смотри математик может взять и за три дня выучить хачкель или лисп по sicp, а потом на нем полезные программы писать. А может ебаться с императивной поеботиной, а потом с ассемблером пару лет. Нахуй надо?
Аноним 01/04/15 Срд 18:37:18 #550 №252718 
>>252525
>>252515
Это не математические рассуждения, чел, по - крайней мере в том виде в котором ты привёл. Попытайся излагать яснее, и раскрыть таки тему что же такое
> множество измерений - бесконечность
> множество трехмерных пространств ... стремиться к бесконечности
> проекция точки
итд.
Аноним 01/04/15 Срд 20:10:56 #551 №252745 
>>252713

Математик может. Но ты хуйло тупое, даже не понял что тебе написали.
Аноним 01/04/15 Срд 20:12:28 #552 №252747 
>>252713

> Боль первокультурника.

Боль в анусе твоей мамки-шлюхи, после того как там побывали мои 20 см.
Аноним 01/04/15 Срд 20:42:52 #553 №252755 
14279101726010.jpg
Пиздос, то раздел лежит неделю когда какие-то задачки конкретные кидают, то начинается бурное обсуждение когда вскрывается какая-то околофилософская глобальная тип тема, которую можно обсуждать без особых знаний и усилий.

Я повторю свой смывшийся вопросик: может-ли кто-то подсказать книжку о конформной теории поля, в которой обстоятельно рассказано про приложения модулярных алгебр Ли и квантовых групп к этому всему делу. С меня как обычно.
sageАноним 01/04/15 Срд 20:56:20 #554 №252766 
http://www.svoboda.org/content/article/26921920.html
Аноним 01/04/15 Срд 21:13:51 #555 №252771 
>>252755
>В конце августа 2012 года японский математик
Новость твоя протухла, уноси.
Аноним 01/04/15 Срд 21:14:45 #556 №252772 
>>252771 -> >>252766

Аноним 01/04/15 Срд 21:26:29 #557 №252777 
>>252766

Пидорские ресурсы не посещаю.
Аноним 01/04/15 Срд 22:37:29 #558 №252820 
>>252766
А если серьёзно, то в чём трудность основная, в объёме пререквезитов? Без журналистской хуйни вроде
> должны деактивировать паттерны сознания
?
Аноним 01/04/15 Срд 23:05:47 #559 №252842 
>>252820
Увы, без деактивации никак, иначе соснёшь.
Аноним 01/04/15 Срд 23:49:25 #560 №252854 
>>252260
Бампую вопрос. Можно ли полностью и до конца решить уравнение теплопроводности не прибегая к численным методам?
Аноним 02/04/15 Чтв 00:01:15 #561 №252856 
>>252854
Смотря для какой геометрии. Для стержня, в шаре, или кубе можно. Для многих двумерных можно конформным преобразованием. Для остальной жопы без численных методов заебешься.
sageАноним 02/04/15 Чтв 00:39:03 #562 №252868 
>>252766
Поцоны. Кароч, проверил. И знаете чо? А заебись. Всё сходится, кароч. Понимаешь?
Аноним 02/04/15 Чтв 01:04:55 #563 №252876 
>>252745
Да нет, я понял, я привел простую аналогию, то что хачкелисту нахуй не всралась императивная поебень, также как и алгебраическому геометру нахуй не всрались основания.
Аноним 02/04/15 Чтв 01:07:22 #564 №252878 
14279260421340.png
Йо пацаны, вы как, смотрю алггеом я знатно смог зафорсить в этом итт треде в свое время, кек.
Но пока мамкины алггеометры визжат про схемы, то мне для одной научной работы понадобилось изучение конкретных противных многоборий, которые заданы через жопу, чьи идеалы никто нихуя не знает, пришлось исследовать их особые точки для разрешения одной проблемы, пиздос. Эти ебучие пучки и их когомологии пришлось ебашить.
А вы тут пока обсуждайте, кто круче, Гротендик или ваша мамка.

Схемоеб
Аноним 02/04/15 Чтв 01:11:02 #565 №252880 
14279262627200.jpg
>>252876
>как и алгебраическому геометру нахуй не вкрались основания.кококок основания ненужны
Кек, а вот и мамкин алгеометр в этом итт треде. Просто признай, что ты даже матлога не знаешь, поэтому и утверждаешь так.
Аноним 02/04/15 Чтв 02:02:02 #566 №252887 
>>249033
а всё же
кто-нибудь пытался
осилить
работы Мочидзуки?
Аноним 02/04/15 Чтв 02:34:25 #567 №252894 
>>252880
>>252876
Хотя всерьез заморачиваться основаниями для алг. геометров профессионально бессмыслено. Я бы все же рекомендовал взглянуть на ту часть теории множеств, которая нужна для формализации некоторых сложных теоретико-категорных построений (аксиома вселенных Гротендика). Как правило интуиции должно хватать, но тем не менее потенциал для проблем вполне есть.
Ну и из логики я бы всем математикам рекомендовал познакомиться с теоремой о компактности логики первого порядка - несложное утверждение с многочисленными следствиями, касающимися многих разделов математики.
Аноним 02/04/15 Чтв 04:07:06 #568 №252897 
>>252878
>я знатно смог зафорсить

Ты не пуп земли, лол.
Аноним 02/04/15 Чтв 09:28:20 #569 №252912 
>>252876

С таким подходом хачкелист будет высирать медленное говно, а не код.

Твоя аналогия говно. Например, по ней же можно сказать, что твоему абстрактному геометру чтобы начать работать не надо изучать основы той же геометрии - нужно взять дайджест результатов, все вызубрить без докозательств и херачить доказательства новых теорем.


Аноним 02/04/15 Чтв 09:30:55 #570 №252913 
>>252894

Нахуй нужны фрагментарные знания, если браться за оснонования, нужно нормальную книжку изучить, а не пару теоремок выдернутых из контекста.
Аноним 02/04/15 Чтв 09:34:05 #571 №252914 
>>252880

Всё верно. Все уёбки выступающие против оснований в этом итт треде нити просто нихуя не петрят в математике и не умеют в мат.логику. Гротендики ебаные.
Аноним 02/04/15 Чтв 09:36:10 #572 №252915 
>>252766

Cвободу ГРОТЕНДИКАМ!
Аноним 02/04/15 Чтв 10:04:26 #573 №252918 
>>252912
Именно так и надо поступать.
Аноним 02/04/15 Чтв 10:10:28 #574 №252920 
>>252918

Подпесалсо.
Аноним 02/04/15 Чтв 10:30:55 #575 №252922 
14279598559880.png
Думать, что любая формальная конструкция, пришедшая в голову пьяному гротендику и послужившая отправной точкой для бесконечной аутичной игры ею в бисер, обязательно найдет свое применение в теорфизике естественным образом (исключая случай искуственного форсинга) - это надо быть очень радикальным пифагорейцем/платонистом.

Только очень малая часть совр. мат. аппарата юзается физиками. Ну, топология, дифф. геометрия, С*-алгебры, группы, кое-что из алгебраич. геометрии, твисторы, супералгебры (причем многое уже было самой физикой и мотивировано, лол). То, что реально работает и зарекомендовало себя. Ну и таки да, есть ряд отмороженных маргинальщиков, которые занимаются ёба-сверхматематизацией физики и выдристывают статейки в духе "а вот давайте-ка этальных когомологий сюда применим, диаграмм и морфизмов навернём и скажем - воооот, а мы тут у мамы математику к физике применяем, а вы говорите,что эти конструкции нинужныы!". (Нет нужды говорить,что 99% этих статеек оседают на днище журналов и arxiv'ов, никем не замеченные.)
Аноним 02/04/15 Чтв 11:22:25 #576 №252932 
>>249033
Внимание, внимание!
Всем математикам двача!
Предлагаю замутить тут тред со своим polymath проектом, по типу тредов про биологическое бессмертие, только с более осмысленной конечной целью.
понимаю, что
этот polymath сам по себе
немножко говно
потому что скорее не theory building,
а
problem solving,
но может взлететь же
каждый из нас в душе же хотел
бы
пописать программок экспериментальных.
Предлагаю начать с просмотра предложений, собственно, выдвинутых для polymath:
http://polymathprojects.org/
ну и с самой вики
http://michaelnielsen.org/polymath1/index.php?title=Main_Page
Может отдельный тред запилить с обсуждением?
Аноним 02/04/15 Чтв 11:24:42 #577 №252935 
Ну или можно попробовать что-нибудь типа семинара, а не полимата, чтобы theory building, только понять бы ещё как это осуществить в формате двача и есть ли для этого аудитория
Аноним 02/04/15 Чтв 12:40:14 #578 №252944 
>>252935
Берешь, создаешь тред, ищешь откликнувшихся. Если аудитория есть, проводишь в треде.
Аноним 02/04/15 Чтв 12:43:19 #579 №252945 
>>252913
Ты опять начинаешь, пиздюк? Или ты думаешь математикам в своих областях знаний мало, надо еще основания? Если алгеометр, можно 2000 страниц Гротендика прочитать, ахуенно полезно будет.
Аноним 02/04/15 Чтв 12:44:49 #580 №252946 
>>252880
>матлога не знаешь
У меня матлог был в виде двухнедельного рассказа на первом курсе. Мне кажется и этого было много.
Аноним 02/04/15 Чтв 12:46:03 #581 №252947 
>>252913
Т.к. их на самом деле можно применять они полезны и в отрыве от широкого контекста (чтобы вообще разобраться все-таки некоторый контекст в любом случае нужен).

Как это может быть не грустно, но зачастую нужны именно фрагментарные знания т.к. целостных сложны и их изучение занимает слишком много времени. Например, до недавнего времени многим инженерам было полезно знать некоторые методы решения диффуров при полной бесполезности хотя бы доказательств корректной работы этих методов. Или например, всем полезны некоторые фрагментарные медицинские знания, но учить всех биохимическим процессам лежащим в основе ряда заболеваний - это абсурд.
Аноним 02/04/15 Чтв 13:53:43 #582 №252960 
Не пытайтесь говорить с любителем оснований как с нормальным человеком. Это же воплощенная в фейлософе реинкарнация переводчика итт. Дайте уже этому треду умереть спокойно, и не кормите поехавшего в следующем.
Аноним 02/04/15 Чтв 14:35:00 #583 №252968 
>>252960

Иди нахуй, хуйло!
Аноним 02/04/15 Чтв 14:35:01 #584 №252969 
Perequat: https://2ch.hk/sci/res/252964.html
Аноним 02/04/15 Чтв 17:29:28 #585 №253045 
>>252969
Бумплимит 1000 нахуя перекат?
Аноним 03/04/15 Птн 10:34:35 #586 №253257 
>>253045

Похуй на перекат, пляшем здесь.
Аноним 12/04/15 Вск 14:31:00 #587 №255963 
>>252878
Схемоеб, дай фейкомыло для связи.
Поспрашиваю у тебя про CS и программки, кажется ты в pr был.
comments powered by Disqus

Отзывы и предложения